Test 3abc

You might also like

Download as pdf or txt
Download as pdf or txt
You are on page 1of 51

Rs.

25

VISION IAS
PT 2021
Test - 3 (3180)
(Geography – I)

PTO for Syllabus


SYLLABUS

Physical Geography of the world


▪ Solar System & the Earth and basics of latitude and longitude
▪ Geomorphology
▪ Climatology
▪ Oceanography
▪ Biogeography
▪ Natural Hazards and Disasters
▪ Continents (Land, Climate, Resources etc.): Asia, Africa, North
America, South America, Antarctica, Europe, and Australia
Physical Geography of India
▪ Introduction: Location, Area and Boundaries
▪ Structure and Relief: Physiographic Divisions
▪ Drainage System
▪ Weather, Climate and Seasons
▪ Soils
▪ Natural Vegetation, Plant and Animal Life
+ Current Affairs (June 2020)

REFERENCES

▪ 11th NCERT Geography: Fundamentals of Physical Geography


▪ 11th NCERT Geography - India Physical Environment
▪ Class IX NCERT: Contemporary India-I
▪ Certificate physical and human geography - Goh Cheng Leong
▪ Orient Longman Atlas or Oxford Atlas
1. Consider the following: 4. Which of the following statements is/are
1. The sun, the moon and the earth are in a correct regarding the igneous rocks?
straight line. 1. They can form beneath as well as on the
2. Moon's orbit is closest to the Earth. surface of the earth.

3. The tidal range is greater than normal. 2. They are devoid of fossils.
3. Quartzite and granite are igneous rocks.
The above given conditions defines which of
Select the correct answer using the code
the following tides?
given below.
(a) Apogean Spring tides
(a) 1 and 2 only
(b) Perihelion Spring tides
(b) 2 and 3 only
(c) Perigean Spring tides
(c) 1 only
(d) Aphelion Spring tides
(d) 1, 2 and 3

2. Which of the following is/are favorable 5. Consider the following statements regarding
condition(s) for the faster decomposition of the Periodic Labour Force Survey:
the organic matter in the soil? 1. It aims to provide labor and employment
1. Lack of oxygen data for both urban and rural India.
2. Detritus rich in nitrogenous substances 2. It is conducted by the National
3. Cold temperatures Statistical Office.
Select the correct answer using the code Which of the statements given above is/are

given below. correct?

(a) 2 only (a) 1 only


(b) 2 only
(b) 1 and 2 only
(c) Both 1 and 2
(c) 2 and 3 only
(d) Neither 1 nor 2
(d) 1, 2 and 3

6. These cool temperate western margins are


3. Which of the following is/are the
under the permanent influence of the
components of Bharatmala Pariyojana?
Westerlies all round the year. They are also
1. Development of economic corridors. regions of much cyclonic activity. Summers
2. Development of border and international are never very warm here. The natural
connectivity roads. vegetation of this climatic type is a
3. Development of port connectivity roads. deciduous forest like oak, elm, ash, birch,
4. Development of expressways beech, poplar, and hornbeam.
Select the correct answer using the code Which of the following climatic types is
given below. described in the passage given above?
(a) 1 and 3 only (a) Mediterranean Climate
(b) 2 and 4 only (b) British Type Climate.

(c) 1, 2, 3 and 4 (c) Steppe Climate


(d) Natal climate
(d) 1, 2 and 3 only
2 www.visionias.in ©Vision IAS
7. Which of the following correctly describes 10. Consider the following statements regarding
the phenomenon of 'Break in the Monsoon'? Latitudes and Longitudes:
(a) A rain deficient situation in any area 1. Latitude is measured as the angular
when the mean annual rainfall is less
distance of a point on the earth's surface
than 75% of the normal rainfall.
north or south of the equator.
(b) When rain fails to occur for one or more
2. Both the latitude and the longitude of a
weeks during the south-west monsoon
period after having rains for a few days. place are measured from the centre of
(c) Bifurcation of monsoon winds the earth.
into Arabian Sea branch and Bay of Which of the statements given above is/are
Bengal branch. correct?
(d) Sudden onset of the moisture-laden (a) 1 only
winds associated with violent thunder (b) 2 only
and lightening.
(c) Both 1 and 2
(d) Neither 1 nor 2
8. Which of the following evidence supports
the Continental Drift Theory proposed by
Alfred Wegner? 11. With reference to the type of mountains,
1. Jig Saw fit of South American and consider the following statements:
African coastlines. 1. Fault mountain involves both the
2. The occurrence of rich placer deposits of compression and tension of the earth's
gold in the Ghana coast despite the crust, while fold mountain involves
absence of source rocks.
mainly compression.
3. Distribution of identical fossils across
2. The Andes is an example of fault
oceans.
mountains whereas the Vosges is an
Select the correct answer using the code
given below. example of fold mountains.
(a) 1 and 2 only Which of the statements given above is/are
(b) 2 and 3 only correct?
(c) 1 and 3 only (a) 1 only
(d) 1, 2 and 3 (b) 2 only
(c) Both 1 and 2
9. Consider the following statements with
(d) Neither 1 nor 2
respect to ocean trenches:
1. They are located along divergent plate
boundaries. 12. In the context of geomorphology, the term
2. They are associated with active playas refer to
volcanoes and strong earthquakes. (a) shallow lakes in the depressions of
3. Tonga trench and Mariana trench are plains found in desert regions.
located in the Pacific Ocean. (b) unassorted coarse and fine debris
Which of the statements given above is/are dropped by melting glaciers.
correct?
(c) large sized angular blocks plucked from
(a) 2 only
land by glaciers.
(b) 1 and 3 only
(d) small round shallow depressions formed
(c) 2 and 3 only
(d) 1, 2 and 3 on limestone surfaces.

3 www.visionias.in ©Vision IAS


13. Which of the following statements is/are 16. Consider the following landforms:
correct with reference to the Western 1. Stalagmites
Disturbances?
2. Sinkholes
1. They are shallow cyclonic depressions
originating over the eastern 3. Uvalas
Mediterranean Sea. 4. Lapies
2. Their arrival in India is characterized by Which of the landforms given above
a sudden drop in the prevailing night
are erosional landforms formed by the
temperature.
3. It is highly beneficial for rabi crops in action of groundwater?
Northern India. (a) 1 and 4 only
Select the correct answer using the code (b) 2 and 3 only
given below.
(c) 2, 3 and 4 only
(a) 1 and 2 only
(b) 2 only (d) 1, 2, 3 and 4
(c) 1 and 3 only
(d) 1, 2 and 3 17. With reference to River Luni, which of the
following statements is correct?
14. With reference to the earth's gravity(g),
consider the following statements: (a) It is formed by two streams, the
1. It is influenced by the distribution of Saraswati and Sabarmati.
mass of material within the earth. (b) It flows through the states of Madhya
2. It is greater near the poles than at the
Pradesh, Rajasthan and Gujarat.
equator.
Which of the statements given above is/are (c) It drains into Gulf of Khambhat.
correct? (d) It is cut twice by the Tropic of Cancer.
(a) 1 only
(b) 2 only
18. Consider the following statements:
(c) Both 1 and 2
(d) Neither 1 nor 2 1. While a lunar eclipse can occur twice a
year, a solar eclipse occurs only once a
15. Which of the following is/are proof(s) of the year.
sphericity of the Earth?
2. A solar eclipse always takes place on the
1. Possible circumnavigation of the earth
summer solstice of the Northern
without a steep fall.
2. Different times of sunrise and sunset at Hemisphere.
different places on earth. Which of the statements given above is/are
3. Shape of earth's shadow during a lunar
correct?
eclipse.
(a) 1 only
Select the correct answer using the code
given below. (b) 2 only
(a) 1 and 2 only (c) Both 1 and 2
(b) 1 and 3 only
(d) Neither 1 nor 2
(c) 2 only
(d) 1, 2 and 3
4 www.visionias.in ©Vision IAS
19. The Western Ghats are locally known by 22. Which of the following are the effects of the
different names. In this context, consider the Himalayan Mountains on Indian climate?
1. It protects the subcontinent from the
following pairs:
cold northern winds.
Local name Region
2. It traps the monsoon winds, forcing them
1. Sahyadri : Maharashtra to shed their moisture within the
2. Nilgiri : Tamil Nadu subcontinent.
3. Anamalai : Karnataka 3. It creates different air pressure zones
Which of the pairs given above is/are causing a reversal in the direction of
monsoon winds.
correctly matched?
Select the correct answer using the code
(a) 1 and 2 only
given below.
(b) 1 and 3 only (a) 1 and 2 only
(c) 1 only (b) 2 and 3 only
(d) 2 and 3 only (c) 1 and 3 only
(d) 1, 2 and 3

20. Which of the following statements is correct


23. In the context of Indian geography, Karewas
in the context of the Lakshadweep Islands? are
(a) The entire island group is built of coral (a) thick deposits of glacial clay and other
deposits. materials found in Kashmir Himalayas.
(b) Narcondam is a small volcanic island in (b) alluvial fans formed by the tributaries of
Indus in Northwest region of India.
Lakshadweep group.
(c) ill-drained, damp and thickly forested
(c) Saddle peak is the highest peak in
narrow tracts at the foothills of
Lakshadweep islands. Himalayas
(d) Eleven degree channel separates (d) oases in the Thar desert.
Lakshadweep from Maldives.
24. It is a unique rice variety in the coastal areas
of Alappuzha, Ernakulam and Thrissur
21. With reference to Longitudes and Standard
districts. The salt water-resistant variety
Time Zones, consider the following grows tall, flourishes under flood-like
statements: conditions and grows entirely on leftovers
1. The entire world is divided into fifteen from half-a-year of fish culture in fields. The
time zones. cycle is called ‘one fish and one rice’ in
which fish is grown up to April 15 after
2. All countries have adopted their standard
which fields are drained before the first bout
time from the central meridian of their
of rains get them rid of salt. The season goes
countries. up to October when they are brought under
Which of the statements given above is/are fish culture again.
correct? Which of the following varieties of rice is
(a) 1 only being described in the above passage?
(a) Pokkali
(b) 2 only
(b) Damodar
(c) Both 1 and 2 (c) Porteresia
(d) Neither 1 nor 2 (d) IR-8
5 www.visionias.in ©Vision IAS
25. Consider the following statements: 28. In which of the following plate boundaries,
1. The Rivers that existed before the new crust formation takes place?
upheaval of the Himalayas and cut their
1. Convergent Boundaries
courses southward by making gorges in
the mountains are known as the 2. Divergent Boundaries
superimposed rivers. 3. Transform Boundaries
2. The Rivers which follow the general Select the correct answer using the code
direction of the slope are known as the
given below.
consequent rivers.
Which of the statements given above is/are (a) 1 and 2 only
correct? (b) 2 only
(a) 1 only
(c) 1 and 3 only
(b) 2 only
(d) 1, 2 and 3
(c) Both 1 and 2
(d) Neither 1 nor 2
29. In the context of Indian physiography,
26. With reference to the structure of the earth,
Malda fault separates
consider the following statements:
1. The outer core is in a liquid state while (a) The Vindhyas from the Satpura ranges.
the inner core is in solid-state (b) The Delhi ridge from the Aravallis.
2. The outer core is the main source of (c) The Eastern Ghats from the Western
magma that finds its way to the surface
Ghats
during volcanic eruptions.
Which of the statements given above is/are (d) Meghalayan plateau from the Peninsular
correct? plateau.
(a) 1 only
(b) 2 only
30. Which of the following weather
(c) Both 1 and 2
(d) Neither 1 nor 2 phenomenon are observed during the winter
season in India?
27. With reference to the difference between 1. Development of high pressure in the
continental and oceanic crusts, consider the
region lying to the north of the
following statements:
1. The main mineral constituents of the Himalayas.
continental mass are silica and 2. The inflow of dry surface winds from
magnesium while the oceanic crust
Central Asia.
mainly consists of silica and aluminium.
3. The onset of an easterly jet stream over
2. The continental crust is composed
mostly of granite while the oceanic crust peninsular India.
mostly consists of a volcanic lava rock Select the correct answer using the code
called basalt.
given below.
Which of the statements given above is/are
correct? (a) 1 and 2 only
(a) 1 only (b) 2 and 3 only
(b) 2 only (c) 1 and 3 only
(c) Both 1 and 2
(d) 1, 2 and 3
(d) Neither 1 nor 2
6 www.visionias.in ©Vision IAS
31. Consider the following statements in the 34. Which of the following vertebrates first
context of Central Highlands (or the Madhya originated in the course of geological time
scale?
Bharat Plateau):
(a) Amphibians
1. The general slope of the highlands is (b) Fish
towards the south and south-east. (c) Reptiles
2. Musi river basin forms the majority part (d) Mammals
of the region.
35. Consider the following pairs regarding
Which of the statements given above is/are
festivals recently seen in news:
correct? Festival State
(a) 1 only 1. Raja Parba : West Bengal
(b) 2 only 2. Thrissur Puram : Kerala
3. Ambubachi Mela : Assam
(c) Both 1 and 2
Which of the pairs given above are correctly
(d) Neither 1 nor 2 matched?
(a) 1 and 2 only
32. Consider the following statements with (b) 2 and 3 only
(c) 1, 2 and 3
reference to Indus River System:
(d) 1 and 3 only
1. The area covered by the Indus River
System is more in Pakistan than India. 36. Consider the following rivers:
2. The basin area of river Indus lies in three 1. Tapi
2. Mahi
countries.
3. Damodar
Which of the statements given above is/are
Which of the rivers given above flow
correct? through rift valleys?
(a) 1 only (a) 1 and 2 only
(b) 2 only (b) 2 and 3 only
(c) 1, 2 and 3
(c) Both 1 and 2
(d) 1 and 3 only
(d) Neither 1 nor 2
37. In the context of the Peninsular plateau,
33. Consider the following local winds: consider the following statements:
1. The general elevation of the Peninsular
1. Sirocco
plateau is from the west to the east.
2. Mistral 2. The plateau is devoid of crustal faults
3. Foehn and fractures.
4. Chinook 3. Shillong and Karbi-Anglong plateau are
the extensions of the Peninsular plateau.
Which of the winds mentioned above blow
Which of the statements given above are
over the Mediterranean region? correct?
(a) 1 and 2 only (a) 1 and 2 only
(b) 2 and 3 only (b) 2 and 3 only
(c) 1, 2 and 3
(c) 3 and 4 only
(d) 1 and 3 only
(d) 1 and 3 only
7 www.visionias.in ©Vision IAS
38. Which of the following is/are the aims of 41. Consider the following statements regarding
National Forest Policy 1988? the Earthquake:
1. Bringing 33 percent of the geographical 1. All the natural earthquakes take place in
area under forest cover. the lithosphere
2. Reduction of floods and droughts. 2. The point where the energy is released is
3. Creating people's movement to called the epicenter.
encourage the planting of trees. Which of the statements given above is/are
Select the correct answer using the code correct?
given below. (a) 1 only
(a) 1 only (b) 2 only
(b) 2 and 3 only
(c) Both 1 and 2
(c) 1 and 3 only
(d) Neither 1 nor 2
(d) 1, 2 and 3

42. Consider the following statements regarding


39. Consider the following statements regarding
the newly launched PM SVANidhi scheme
the Solar System:
for the street vendors:
1. All the planets have more than one
1. It aims to facilitate interest-free loans to
natural satellite.
2. All the planets rotate on their own axis the street vendors for resuming their
in the west to east direction. livelihoods activities, after easing of
3. Saturn is the only planet to have a ring lockdown.
system around itself. 2. No collateral security is required to avail
Which of the statements given above is/are the loan
correct? 3. It is launched by the Ministry of
(a) 1 only Finance.
(b) 2 and 3 only Which of the statements given above is/are
(c) 1 and 3 only correct?
(d) None (a) 1 and 3 only
(b) 2 and 3 only
40. With reference to the State Food Safety (c) 2 only
Index, to rank, the states of India on the (d) 1, 2 and 3
safety of food was released. In this context,
consider the following statements: 43. Which of the following are not a major
1. It is an index released by the Food tectonic plate?
Safety and Standards Authority of India 1. Antarctica plate
(FSSAI). 2. Cocos plate
2. States are adjudged on the parameters of 3. Arabian plate
'Compliance' as well as 'Consumer
4. South American plate
empowerment'.
Select the correct answer using the code
Which of the statements given above is/are
given below.
correct?
(a) 1, 2 and 4 only
(a) 1 only
(b) 2 and 3 only
(b) 2 only
(c) 3 and 4 only
(c) Both 1 and 2
(d) 2, 3 and 4 only
(d) Neither 1 nor 2
8 www.visionias.in ©Vision IAS
57. Consider the following statements with 60. Known to be the land of rolling mountains,
respect to ocean currents: the region is accompanied by the formation
1. West coasts of the continents in the of foreland basins (valley like depressions)
middle latitudes are bordered by cold which run parallel to the mountains, these
currents leading to an arid climate. depression are accumulated vastly with soft
2. The Coriolis force causes the water to unconsolidated deposits forming a molasses
move to the right in the northern basin.
hemisphere and to the left in the Which Indian state is being described in the
southern hemisphere above passage?
Which of the statements given above is/are (a) Assam
correct? (b) Manipur
(a) 1 only (c) Himachal Pradesh
(b) 2 only (d) Mizoram
(c) Both 1 and 2
(d) Neither 1 nor 2 61. Which of the following conditions is/are
responsible for the formation of a China type
58. With reference to the recently launched of climate?
Indian Gas Exchange, consider the following 1. Intense heating in the continental
statements: interiors of Asia in summers.
1. It is the first nationwide online delivery- 2. The steep pressure gradient between the
based gas trading platform in India. cold interiors of Mongolia and Siberia,
2. It will facilitate market-driven price and the warmer Pacific coastlands in
determination of gas in India. winters.
3. It has been incorporated by the National Select the correct answer using the code
Commodity and Derivatives Exchange. given below.
Which of the statements given above is/are (a) 1 only
correct? (b) 2 only
(a) 1 and 2 only (c) Both 1 and 2
(b) 2 and 3 only (d) Neither 1 nor 2
(c) 1 only
(d) 1, 2 and 3 62. Which of the following statements correctly
describes Agro-forestry?
59. The biennial Environmental Performance (a) Raising trees on public land such as
Index report is published by pasture and temple land, roadside, etc.
(a) Cornell University in collaboration with (b) Raising and management of trees on
United Nations Environment Programme public and private land.
(b) Yale University and Columbia (c) Raising of trees for commercial and non-
University in collaboration with World commercial purposes on their farm
Economic Forum lands.
(c) International Union for Conservation of (d) Raising of trees and agriculture crops on
Nature (IUCN) the same land inclusive of the waste
(d) World Wildlife Fund (WWF) patches.
11 www.visionias.in ©Vision IAS
63. These are smooth oval-shaped ridge-like 66. Recently India was elected as a non-
depositional features composed mainly of permanent member of the United Nations
glacial till with some masses of gravel and Security Council. Which of the following
sand. One end of them facing the glacier countries were elected along with India?
called the stoss end is blunter and steeper 1. Ireland
than the other end called the tail. They give 2. Mexico
an indication of the direction of the glacier 3. Canada
movement. 4. Norway
Which of the following landforms is
Select the correct answer using the code
described in the passage given above?
given below.
(a) Moraines
(a) 1, 2 and 3 only
(b) Eskers
(b) 1, 2 and 4 only
(c) Drumlins
(c) 3 and 4 only
(d) Serrated ridges
(d) 1, 2, 3 and 4

64. Consider the following statements:


1. The Western Ghats are comparatively 67. Arrange the following regions in increasing
higher in elevation to the Eastern Ghats. order of annual rainfall variability.
2. The Western Ghats are less continuous 1. Western Ghats
than the Eastern Ghats. 2. Western Rajasthan
3. The average height of Western Ghats 3. Western Gangetic Plains
increases from north to south. Select the correct answer using the code
Which of the statements given above are given below.
correct? (a) 1-2-3
(a) 1, 2 and 3 (b) 1-3-2
(b) 1 and 2 only (c) 3-2-1
(c) 1 and 3 only (d) 2-1-3
(d) 2 and 3 only
68. Consider the following statements with
65. With reference to the Western Coastal regard to River Vamsadhara:
plains, which of the following statements
1. It is an east-flowing river between
is/are correct?
Rushikulya and Godavari.
1. They are spread out in five Indian states.
2. The river passes through Andhra
2. They are broader in the middle and get
Pradesh and Odisha.
narrower towards North and South.
3. Boddepalli Rajagopala Rao Project was
3. The Malabar stretch of the western
constructed on it to meet the irrigation
coastal plains has backwaters called
needs.
Kayals.
Select the correct answer using the code Which of the statements given above is/ are

given below. correct?


(a) 1 only (a) 1 only
(b) 1 and 3 only (b) 1 and 3 only
(c) 2 only (c) 2 and 3 only
(d) 3 only (d) 1, 2 and 3

12 www.visionias.in ©Vision IAS


69. With reference to the Seafloor spreading 72. Red and Yellow soils are widely prevalent in
thesis, consider the following statements: the Deccan Plateau. With reference to these
1. Constant eruptions at the crest of soils, consider the following statements:
oceanic ridges create new plate 1. They develop on crystalline igneous
material. rocks in areas of low rainfall.
2. It is verified by the age of rocks as rocks 2. The reddish color of the soil is due to the
diffusion of Iron.
closer to mid-oceanic ridge are the
Which of the statements given above is/are
oldest.
correct?
Which of the statements given above is/are
(a) 1 only
correct?
(b) 2 only
(a) 1 only (c) Both 1 and 2
(b) 2 only (d) Neither 1 nor 2
(c) Both 1 and 2
(d) Neither 1 nor 2 73. Consider the following statements with
respect to continental shelf, a division of
70. Which of the following geological events ocean floor:
has shaped the present drainage system of 1. It is the shallowest part of the ocean.
peninsular India? 2. The width of the continental shelves
1. Submergence of the western flank of the vary from one ocean to another.
peninsula. 3. Rich fishing grounds are located in
continental shelves due to the
2. Upheavals of the Himalayas.
availability of planktons.
3. Slight tilting of the Peninsular block
Which of the statements given above are
from northwest to the southeastern
correct?
direction.
(a) 1 and 2 only
Select the correct answer using the code (b) 2 and 3 only
given below. (c) 1 and 3 only
(a) 1 and 2 only (d) 1, 2 and 3
(b) 2 and 3 only
(c) 1 and 3 only 74. Consider the following statements regarding
(d) 1, 2 and 3 forms of precipitation:
1. Snow is formed when the temperature is
71. Which of the following are the outcomes of lower than 0-degree celsius and moisture
exogenic processes? is released in the form of hexagonal
1. Denuded Hills crystals
2. Hailstones is formed when rainwater
2. Earthquakes
passes through successive layers of
3. Fold Mountains
colder layers and forms several
4. Soil creep
concentric layers of ice.
Select the correct answer using the code
Which of the statements given above is/are
given below.
correct?
(a) 1, 2 and 3 only (a) 1 only
(b) 2 and 3 only (b) 2 only
(c) 1 and 4 only (c) Both 1 and 2
(d) 1, 2, 3 and 4 (d) Neither 1 nor 2
13 www.visionias.in ©Vision IAS
75. Which of the following are Atlantic Ocean 78. Consider the following statements regarding
currents? black soil:
1. Benguela current 1. It is widespread in northern and eastern
2. Canary current plains.
3. Alaska current 2. It is generally clayey, deep and
4. Humboldt current impermeable.
5. Labrador current 3. They are also known as regur soil.
Select the correct answer using the code Which of the statements given above is/are
given below. correct?
(a) 1, 2 and 5 only (a) 1 and 3 only
(b) 2, 4 and 5 only (b) 2 and 3 only
(c) 1, 3 and 5 only (c) 3 only
(d) 1, 2, 3 and 4 only (d) 2 only

76. The rise and fall of the tides play an 79. Consider the following:
important role in the natural world and can 1. Rift valley lakes
have a marked effect on maritime-related 2. Tarn lakes
activities. In this context, the tides 3. Kettle lakes
1. help in removing of polluted water from Which of the lakes given above are formed
estuaries. due to glaciation?
2. enable ships to arrive at the harbour in (a) 1 and 2 only
the shallow water safely. (b) 2 and 3 only
3. can be used to generate electricity. (c) 1 and 3 only
4. help in fishing. (d) 1, 2 and 3
Select the correct answer using code given
below. 80. Which of the following causes the complete
(a) 1 and 4 only reversal of North Indian Ocean currents
(b) 2 and 3 only between summer and winter?
(c) 2, 3 and 4 only (a) Coriolis force
(d) 1, 2, 3 and 4 (b) Thermohaline circulation
(c) Movement of counter equatorial current
77. Debris avalanches and landslides occur very (d) Monsoon winds
frequently in the Himalayas because:
1. They are tectonically active. 81. Which of the following are the direct sources
2. They are mostly made up of information about the Earth's Interior?
of unconsolidated and semi-consolidated 1. Rocks from mining areas
deposits. 2. Meteors
3. They receive heavy rainfall over short 3. Seismic activity
periods. 4. Volcanic eruption
Select the correct answer using the code Select the correct answer using the code
given below. given below.
(a) 1 only (a) 1, 2 and 3 only
(b) 1 and 2 only (b) 2 and 3 only
(c) 2 and 3 only (c) 1 and 4 only
(d) 1, 2 and 3 (d) 1, 2, 3 and 4

14 www.visionias.in ©Vision IAS


82. Which of the following UNESCO Biosphere 86. Which of the following countries have
Reserve is located between the Narmada and neither ratified nor signed the Arms Trade
the Tapi river valleys? Treaty?
(a) Pachmarhi 1. India
(b) Achanakmar-Amarkantak 2. Germany
(c) Agasthyamalai 3. China
(d) Similipal 4. Lebanon
5. North Korea
83. Which of the following statements is correct Select the correct answer using the code
with respect to the salinity in the oceans? given below.

(a) Salinity in equatorial waters is higher (a) 1, 2 and 4 only


(b) 1, 2, 3 and 4 only
than the average ocean water salinity.
(c) 2 and 4 only
(b) In the zone of Halocline, salinity rapidly
(d) 1, 3 and 5 only
decreases with increasing depth.
(c) Sodium Chloride constitutes more than
87. With reference to the types of Earthquake
75% of the dissolved mineral water in
Waves, consider the following statements:
the oceans.
1. Primary waves move faster than the
(d) Arabian Sea shows lower salinity trends
secondary waves.
than Bay of Bengal.
2. Primary waves can travel only through
solid materials while secondary waves
84. Which of the following is/are the impacts of
can travel through any medium.
occurrence of El-Nino?
Which of the statements given above is/are
1. Distortion of equatorial atmospheric
correct?
circulation. (a) 1 only
2. Delay in the onset of southwest (b) 2 only
monsoon over the Indian-subcontinent. (c) Both 1 and 2
3. Reduction in the number of fish in the (d) Neither 1 nor 2
sea along the Peruvian coast.
Select the correct answer using the code 88. Consider the following statements with
given below. reference to River Yamuna:
(a) 1 and 2 only 1. It is the westernmost tributary of River
(b) 2 only Ganga.
(c) 2 and 3 only 2. It has its source on the western slopes of
(d) 1, 2 and 3 Banderpunch range in Uttarakhand.
3. It passes through four states and one
85. Which of the following regions can be best Union territories of India.
described by the term "Bush-veld" Which of the statements given above is/ are
landscape? correct?
(a) Steppe grasslands (a) 1 and 3 only
(b) Mediterranean region (b) 2 and only
(c) Savannah grasslands (c) 1 and 2 only
(d) Hot deserts (d) 1, 2 and 3
15 www.visionias.in ©Vision IAS
89. These are dome-shaped granitic bodies 92. Consider the following statements regarding
formed from the cooled portion of magma the Asian Infrastructure Investment Bank
chambers. They appear on the surface only (AIIB):
after the denudational processes remove the 1. Membership to AIIB is limited to Asian
overlying materials. They cover large areas, countries only.
and at times, assume depth that may be 2. It finances projects within Asia only.
several kilometers. 3. India is one of its founding members.
Which of the following volcanic landform is Which of the statements given above is/are
described in the above passage? correct?
(a) Phacolith (a) 1 and 2 only
(b) Lapolith (b) 2 and 3 only
(c) Dyke (c) 3 only
(d) Batholiths (d) 1, 2 and 3

90. Consider the following statements in the 93. Winter monsoons do not cause rainfall in
context of Himalayas: most parts of India because:
1. They are of tectonic origin. 1. They do not have adequate humidity to
2. These are the world's youngest folded cause rainfall.
mountains. 2. Anticyclonic circulation on land reduces
3. There is no volcanic activity in the the possibility of rainfall.
Himalayan region. Select the correct answer using the code
Which of the statements given above are given below.
correct? (a) 1 only
(a) 1, 2 and 3 (b) 2 only
(b) 1 and 2 only (c) Both 1 and 2
(c) 2 and 3 only (d) Neither 1 nor 2
(d) 1 and 3 only
94. “This type of climate is experienced along
91. Consider the following pairs: the eastern coasts of tropical lands, receiving
Range/Hill in news State/UT steady rainfall from the Trade Winds all the
1. Saser Kangri : Himachal Pradesh time. The rainfall is both orographic as well
2. Lali Ane : Arunachal Pradesh as convectional. There is no month without
3. Chang Chenmo : Ladakh any rainfall.”
Which of the pairs given above is/are Which type of climate is being referred to in
correctly matched? the above passage?
(a) 1 and 2 only (a) The Hot, Wet Equatorial climate.
(b) 2 and 3 only (b) Tropical Marine Climate
(c) 1 only (c) Tropical Monsoon Climate
(d) 1, 2 and 3 (d) The Sudan Climate

16 www.visionias.in ©Vision IAS


95. Consider the following statements: 98. Consider the following pairs:
1. All parts of Southern India receive Water Channels Between
rainfall from Northeast monsoons. 1. 10 degrees : Andaman and
2. Northern India experiences a high daily Channel Nicobar Islands
and annual range of temperatures than 2. Duncan : Middle Andaman
Southern India. passage and Little Andaman
Which of the statements given above is/are 3. Coco Strait : Great Nicobar and
correct? Sumatran Island
(a) 1 only Which of the pairs given above is/are
(b) 2 only correctly matched?
(c) Both 1 and 2 (a) 1 and 2 only
(d) Neither 1 nor 2 (b) 2 and 3 only
(c) 1 and 3 only
96. Consider the following pairs: (d) 1 only
Desert Country
1. Patagonian Desert : Saudi Arabia 99. Residual mountains are formed as a result of
2. Mojave Desert : United States of erosion of an existing elevated area. Which
America of the following hills are residual hills or
3. Gibson Desert : Turkey mountains?
Which of the pairs given above is/are 1. Aravallis
correctly matched? 2. Satpura hills
(a) 1 only 3. Javadi hills
(b) 1 and 3 only Select the correct answer using the code
(c) 2 only given below.
(d) 1, 2 and 3 (a) 2 and 3 only
(b) 1 and 2 only
97. Which of the following process of the rock (c) 1 and 3 only
formation is being described in the below (d) 1, 2 and 3
passage?
When the rocks of the earth’s surface area 100. With reference to the Inter-Tropical
exposed to denudational agents, they are Convergence Zone (ITCZ), consider the
broken up into various sizes of fragments. following statements:
Such fragments are transported by different 1. It is a low-pressure zone formed by the
exogenous agencies and deposited. These interaction of trade winds and westerlies
deposits through compaction turn into in the tropics.
rocks. It is a process of porosity destruction 2. Its oscillation causes annual variation in
through compaction and fermentation. intensity and the amount of rainfall.
Select the correct answer using the code Which of the statements given above is/are
given below. correct?
(a) Recrystallisation (a) 1 only
(b) Foliation (b) 2 only
(c) Lithification (c) Both 1 and 2
(d) Exfoliation (d) Neither 1 nor 2
VISION IAS
www.visionias.in
ANSWERS & EXPLANATIONS
GENERAL STUDIES (P) TEST – 3180 (2021)

Q 1.C
• The periodical rise and fall of the sea level, once or twice a day, mainly due to the attraction of the sun
and the moon, is called a tide. The moon’s gravitational pull to a great extent and to a lesser extent
the sun’s gravitational pull, are the major causes for the occurrence of tides. Another factor
is centrifugal force, which is the force that acts to counter the balance the gravity. Together, the
gravitational pull and the centrifugal force are responsible for creating the two major tidal bulges on the
earth.
• Spring tides: The position of both the sun and the moon in relation to the earth has direct bearing on tide
height. When the sun, the moon and the earth are in a straight line, the height of the tide will be
higher. These are called spring tides and they occur twice a month, one on full moon period and another
during new moon period.
• Once in a month, when the moon’s orbit is closest to the earth (perigee), unusually high and low tides
occur. During this time the tidal range is greater than normal. Two weeks later, when the moon is
farthest from earth (apogee), the moon’s gravitational force is limited and the tidal ranges are less than
their average heights.
• If a lunar perigee falls near new or full Moon (spring tides occur), then perigean spring tides of greater
tidal range occur. Hence option (c) is the correct answer.
Q 2.A
• Many factors affects decomposition of organic matter.
o Moisture: Maximum decomposition occurs at the intermediate level of moisture in soil. Soil gas
exchange will be limited in high moisture soil leading to low oxygen concentrations and potentially
anaerobic conditions. Also, low moisture content makes the lack of water thus limiting the microbial
metabolism. Hence, both high and low moisture (dry) conditions inhibit the decomposition
process.
o Oxygen: It accelerates the process of decomposition. Since major organisms depend on the oxygen for
respiration and reproduction, they can not multiply in hypoxic conditions i.e. lack of enough oxygen
levels. Hence option 1 is not correct.
o Composition: Decomposition will be faster if the detritus is rich in nitrogen and water-soluble
substances like sugars and it will be slower if the detritus is rich in lignin and chitin. Hence option 2
is correct.
o Temperature: Cool temperatures inhibit the decomposition and piling up of the organic content in
the soil takes place. Warm temperature makes the microorganisms active and faster will be the
decomposition. Hence option 3 is not correct.
o The soil pH can also influence plant growth by its effect on activity of beneficial microorganisms.
Bacteria (like Rhizobia) that decompose soil organic matter are hindered in strong acid soils.
Q 3.C
• Recent Context: According to rating agency ICRA the first phase of the ambitious Bharatmala
Pariyojana (BMP) that was scheduled for completion in 2021-22 is now likely to get completed by 2025-
26, rating agency ICRA
• Bharatmala Pariyojana is an umbrella project for the highways sector with Phase I from 2017 to
2022. A total of around 24,800 km are being considered in Phase I. In addition, Phase I also includes
10,000 km of balance road works under National Highways Development Project. Bharatmala Pariyojana
subsume all existing highway projects including the flagship National Highways Development Project
(NHDP), launched in 1998. Bharatmala Pariyojana consists of 6 components:
o Economic corridors development
o Inter-corridor & feeder roads
1 www.visionias.in ©Vision IAS
o National Corridors Efficiency improvements:
o Border & International connectivity roads
o Coastal & port connectivity roads
o Expressways
• Additional Information-About ICRA:
o ICRA Limited formerly Investment Information and Credit Rating Agency of India Limited was set
up in 1991 by leading financial/investment institutions, commercial banks, and financial services
companies as an independent and professional investment Information and Credit Rating Agency. The
international Credit Rating Agency Moody’s Investors Service is ICRA’s largest shareholder).
Q 4.A
• There are many different kinds of rocks that are grouped under three families on the basis of their mode of
formation. They are:
• Igneous Rocks: Igneous rocks form out of magma and lava from the interior of the earth, so they are
known as primary rocks. The igneous rocks (Ignis – in Latin means ‘Fire’) are formed when magma cools
and solidifies. When magma in its upward movement cools and turns into a solid form it is called igneous
rock. The process of cooling and solidification can happen in the earth’s crust (plutonic rocks) or on
the surface of the earth (volcanic rocks). Hence, statement 1 is correct.
• Igneous rocks are devoid of any fossils. This is because any fossils in the original rock will have melted
when the rock melted to form magma. Hence, statement 2 is correct.
• Igneous rocks are classified based on texture. The texture depends upon the size and arrangement of
grains or other physical conditions of the materials. If molten material is cooled slowly at great depths,
mineral grains may be very large. Sudden cooling (at the surface) results in small and smooth grains.
Intermediate conditions of cooling would result in intermediate sizes of grains making up igneous rocks.
• Granite, gabbro, pegmatite, basalt, volcanic breccia, and tuff are some of the examples of igneous rocks.
• Metamorphic Rocks: They are formed out of existing rocks undergoing recrystallization. Gneissoid,
granite, syenite, slate, schist, marble, quartzite etc. are some examples of metamorphic rocks. Hence,
statement 3 is not correct.
Q 5.C
• Recently, the National Statistical Office (NSO) released the Periodic Labour Force Survey (PLFS)
for July 2018 to June 2019. (National Sample Survey Office (NSSO) is a part of NSO)
• PLFS is India’s first computer-based survey launched by the National Statistical Office (NSO) in
2017. The PLFS has been launched with an objective of measuring quarterly changes of various labour
market statistical indicators in urban areas as well as generating annual estimates of these indicators both
for rural and urban areas, which can be used for policymaking. It has been constituted based on the
recommendation of Amitabh Kundu.
• PLFS has two fold objective:
o To estimate the key employment and unemployment indicators (viz. Worker Population Ratio, Labour
Force Participation Rate, Unemployment Rate) in the short time interval of three months for the urban
areas only in the Current Weekly Status (CWS).
o To estimate employment and unemployment indicators in both usual status and CWS in both
rural and urban areas annually.
• Periodic Labour Force Survey (PLFS), 2017 is released with changes in survey methodology, data
collection mechanism and sampling design vis-à-vis the earlier quinquennial (once in every five years)
Employment and Unemployment surveys of NSSO.
• Key Highlights:
o Unemployment Rate: India’s unemployment rate fell to 5.8% during 2018-19 from 6.1% during the
same period of 2017-18.
✓ The urban unemployment rate reduced to 7.7% from 7.8%.
✓ The rural unemployment reduced to 5% from 5.3%.
o Labour Force Participation Rate: The labour force participation rate rose to 37.5% during 2018-19
from 36.9% of 2017-18.
✓ Female Participation Rate: The female participation rate also improved going up to 18.6% in
2018-19 from 17.5% the year before.
o Worker Population Ratio: The worker population ratio also increased, to 35.3% as against 34.7% in
2017-18.
Q 6.B
• The Cool Temperate Western Margin (British Type) Climate:
o The cool temperate western margins are under the permanent influence of the Westerlies all round
the year.
2 www.visionias.in ©Vision IAS
o They are also regions of much cyclonic activity, typical of Britain, and are hence named British type
of climate.
• Spatial spread:
o Britain, lowlands North-West Europe, northern and western France, Belgium, the Netherlands,
Denmark, western Norway and also north-western Iberia.
o In the southern hemisphere, southern Chile, Tasmania, and most parts of New Zealand.
• Temperature:
o The mean annual temperatures are usually between 5°C and 15°C.
o The annual range of temperature is small. Summers are, in fact, never very warm.
• Precipitation:
o The British type of climate has adequate rainfall throughout the year with a tendency towards
a slight winter or autumn maximum from cyclonic sources
o Since the rain-bearing winds come from the west, the western margins have the heaviest
rainfall. The amount decreases eastwards with increasing distance from the sea.
• Natural Vegetation:
o The natural vegetation of this climatic type is deciduous forest. The trees shed their leaves in the cold
season, for protecting themselves against the winter snow and frost.
o The common species include oak, elm, ash, birch, beech, poplar, and hornbeam.
o Unlike the equatorial forests, the deciduous trees occur in pure stands and have greater
lumbering value from the commercial point of view.
o The deciduous hardwoods are excellent for both fuel and industrial purposes.
• Economy:
o The region differs from many others in its unprecedented industrial advancement.
• The distinguishing features of the other three types of climate
o Mediterranean Type: It is a warm temperate western-margin climate. It rains here in the winter,
and the summers are dry and warm.
o Steppe Type: It is a temperate continental type of climate with little rainfall and the natural
vegetation is grass (varying in density and quality).
o Natal Type: This is experienced in the warm temperate eastern margin (non-monsoonal areas) of
the southern hemisphere.
• Hence, option (b) is the correct answer.
Q 7.B
• When the trade winds approach the land, their southwesterly direction is modified by the relief and
thermal low pressure over northwest India. The monsoon approaches the landmass in two branches i.e
Arabian Sea branch & Bay of Bengal branch.
• Break/Burst of the Monsoon
o The rain in the southwest monsoon season begins rather abruptly. One result of the first rain is that it
brings down the temperature substantially. This sudden onset of the moisture-laden winds
associated with violent thunder and lightning is often termed as the “break” or “burst” of the
monsoon
• Break in the Monsoon
o If rain fails to occur for one or more weeks during the south-west monsoon period after having
rained for a few days.
o These breaks in the different regions are due to different reasons:
✓ In northern India, rains are likely to fail if the rain-bearing storms are not very frequent along the
monsoon trough or the ITCZ over this region.
✓ Over the west coast, the dry spells are associated with days when winds blow parallel to the coast.
Q 8.D
• Evidence in Support of the Continental Drift:
o The shorelines of Africa and South America facing each other have a remarkable and unmistakable
match. It may be noted that a map produced using a computer programme to find the best fit of the
Atlantic margin was presented by Bullard in 1964. It proved to be quite perfect. The match was tried
at 1,000- fathom line instead of the present shoreline.
o The radiometric dating methods developed in the recent period have facilitated correlating the rock
formation from different continents across the vast ocean. The belt of ancient rocks of 2,000 million
years from Brazil coast matches with those from western Africa. The earliest marine deposits along
the coastline of South America and Africa are of the Jurassic age. This suggests that the ocean did not
exist prior to that time.
3 www.visionias.in ©Vision IAS
o The occurrence of rich placer deposits of gold in the Ghana coast and the absolute absence of
source rock in the region is an amazing fact. The gold bearing veins are in Brazil and it is obvious
that the gold deposits of the Ghana are derived from the Brazil plateau when the two continents lay
side by side.
o When identical species of plants and animals adapted to living on land or in fresh water
are found on either side of the marine barriers, a problem arises regarding accounting for
such distribution. The observations that Lemurs occur in India, Madagascar and Africa led some to
consider a contiguous landmass “Lemuria” linking these three landmasses. Mesosaurus was a small
reptile adapted to shallow brackish water. The skeletons of these are found only in two localities: the
Southern Cape province of South Africa and Iraver formations of Brazil. The two localities presently
are 4,800 km apart with an ocean in between them.
Q 9.C
• Ocean trenches are long, narrow depressions on the seafloor. They are the deepest parts of the oceans.
The trenches are relatively steep sided, narrow basins. They are some 3-5 km deeper than the surrounding
ocean floor.
• Ocean trenches are a result of tectonic activity, which describes the movement of the Earth’s lithosphere.
In particular, ocean trenches are a feature of convergent plate boundaries, where two or more tectonic
plates meet. At many convergent plate boundaries, dense lithosphere melts or slides beneath less-dense
lithosphere in a process called subduction, creating a trench. Hence statement 1 is not correct.
• They occur at the bases of continental slopes and along island arcs and are associated with active
volcanoes and strong earthquakes. That is why they are very significant in the study of plate
movements. As many as 57 deeps have been explored so far; of which 32 are in the Pacific Ocean; 19 in
the Atlantic Ocean and 6 in the Indian Ocean. Hence statement 2 is correct.
• The deepest known trench is Mariana trench near Guam Island which is more than 36,000 feet deep.
Other notable ocean deeps include Tonga trench, Japanese trench, Mindanao deep etc., all located in
Pacific Ocean. Hence statement 3 is correct.
Q 10.C
• Statement 1 is correct: Latitude can be thought of as the angular distance of a point (on the surface of
earth) north or south of the equator. Since the plane of equator is the reference plane for measuring
latitudes, the equator is at 0° latitude. It is parallel to a line, the equator, which lies midway between the
poles. These lines are therefore called parallels of latitude, and on a globe are actually circles, becoming
smaller polewards. The equator represents 0o and the North and South Poles are 90oN and 90oS.
• Statement 2 is correct: Longitude is an angular distance, measured in degrees along the equator east or
west of the prime meridian. On the globe longitude is shown as a series of semi-circles that run from pole
to pole passing through the equator. Such lines are also called Meridians.

Q 11.A
• Mountains make up a large proportion of the earth's surface. Based on their mode of formation, four main
types of mountains are classified:
• Fold Mountains:
o These mountains are by far the most widespread type of mountain.
o They are caused by large scale earth movement when stress is set up in the earth's crust. Such stress
may be due to the increased load of the overlying rocks, flow movement in the mantle, magnetic
intrusions into the crust, or the expansion or contraction of some part of the earth.
4 www.visionias.in ©Vision IAS
o When such stress is initiated, the rocks are subjected to a compressive force that produces
wrinkling or folding along the lines of weakness. Folding effectively shortens the earth's crust,
creating from the original level surface a series of waves. The unfolded waves are called anticlines
and the troughs or downfolds are synclines.
o The fold Mountains are closely associated with volcanic activity. They contain many active
volcanoes, especially in the circum-pacific fold mountain system. They also contain rich mineral
resources such as tin, copper.
o Some of the examples are the Himalayas, Andes, Alps, etc.
• Block Mountain/Fault Mountain
o Block mountains are created because of faulting on a large scale (when large areas or blocks of earth
are broken and displaced vertically or horizontally).
o The uplifted blocks are termed as horsts, and the lowered blocks are called graben.
o Block mountains are also called fault-block mountains since they are formed due to faulting as a
result of tensile and compressive forces.
o The Great African Rift Valley (valley floor is graben), The Rhine Valley (graben), and the Vosges
mountain (horst) in Europe are examples.
• Volcanic Mountain
o A volcanic mountain starts out as a simple crack in the Earth called a volcanic vent. Magma erupts out
of the ground as lava flows, clouds of ash, and explosions of rock. This material falls back to Earth
around the vent, and piles up around it. Overtime (and sometimes quite quickly) a volcanic mountain
builds up. Example Mount Fuji.
• Residual Mountains
o Mountains that have been eroded by the agents of degradation such as winds, rain, frost and running
water. The hard rocks that are left behind are called residual mountains. Example Sierras in Spain and
Mesas in USA.
• Hence only statement 1 is correct.
Q 12.A
• Plains are by far the most prominent landforms in the deserts. In basins with mountains and hills around
and along, the drainage is towards the center of the basin and due to gradual deposition of sediment from
basin margins, a nearly level plain forms at the center of the basin.
• Sometimes the water collected in the depression does not completely disappear by evaporation or
seepage covering the depression or plain with a shallow water body. Such types of shallow lakes are
called playas where water is retained only for a short duration due to evaporation and quite often the
playas contain heavy deposition of salts. Thus these are landforms formed by the action of water in
deserts. The playa plain covered up by salts is called alkali flats.
• Small round shallow depressions formed on limestone surfaces through solution are called Swallow
holes, a common erosional feature of karst topography.
• Unassorted coarse and fine debris dropped by melting glaciers is called Glacial till.
• Hence option (a) is the correct answer.
Q 13.C
• Western Disturbances
o They are shallow cyclonic depressions (weak temperate cyclones) originating over the eastern
Mediterranean Sea and travelling eastwards across West Asia, Iran, Afghanistan and Pakistan before
they reach the northwestern parts of India. Hence, statement 1 is correct.
o They are steered in India by Westerly Jet Streams
o On their way, the moisture content gets augmented from the Caspian Sea in the north and the Persian
Gulf in the south.
o Although the amount of rainfall caused by them is meagre, it is highly beneficial for rabi
crops. Hence, statement 3 is correct.
o It sustains the flow of water in the Himalayan rivers during the summer months.
o An increase in the prevailing night temperature generally indicates an advance in the arrival of these
cyclones disturbances. Hence, statement 2 is not correct.
Q 14.C
• The gravitation force (g) is not the same at different latitudes on the surface.
• The reading of the gravity at different places is influenced by many other factors including the uneven
distribution of mass of material within the earth influences this value.
• The gravity values also differ according to the mass of material.
• It is greater near the poles and less at the equator. This is because of the distance from the centre at the
equator being greater than that at the poles.
5 www.visionias.in ©Vision IAS
• These readings differ from the expected values. Such a difference is called gravity anomaly. Gravity
anomalies give us information about the distribution of mass of the material in the crust of the earth.
• Hence both the statements are correct.
Q 15.D
• All observations from telescopes reveal that the planetary bodies, the Sun, Moon, satellites and stars have
circular outlines from whichever angle they are seen. They are strictly spheres. Earth, by analogy, cannot
be the only exception. The first voyage around the world by Ferdinand Magellan and his crew, from 1519
to 1522 proved beyond doubt that the Earth is a sphere. No traveller going round the world by land or sea
has encountered an abrupt edge, over which he would fall. Hence, statement 1 is correct.
• The distant horizon viewed from the deck of a ship at sea, or from a cliff on land always and everywhere
is circular in shape. This circular horizon widens with increasing altitude and could only be seen on a
spherical body.
• The sunsets and rises at different times at different places. As the earth rotates from west to east, places in
east see the sun earlier than those in the west. If the earth were flat, the whole world would have sunrise
and sunset at the same time. Hence, statement 2 is correct.
• During the lunar eclipse, the shadow cast by the earth on the moon is always circular. It takes the outline
of an arc of a circle. Only a sphere can cast such a circular shadow. Hence, statement 3 is correct.
Q 16.C
• Physical or mechanical removal of materials by moving groundwater is insignificant in developing
landforms. That is why the results of the work of groundwater cannot be seen in all types of rocks.
• But in rocks like limestones or dolomites rich in calcium carbonate, the surface water as well as
groundwater through the chemical process of solution and precipitation deposition develop varieties of
landforms.
• Any limestone or dolomitic region showing typical landforms produced by the action of groundwater is
called Karst topography after the typical topography developed in limestone rocks of Karst region in
the Balkans adjacent to Adriatic sea.
• The karst topography is also characterized by erosional landforms:
o Swallow holes: also known as Sinkholes are funnel-shaped shallow depressions formed on the
surface of limestones through solution.
o Doline: refers to the collapse sinks
o Uvalas: When sinkholes and dolines join together because of slumping of materials along their
margins or due to roof collapse of caves, long, narrow to wide trenches called valley sinks
or Uvalas form.
o lapies: are the ridges formed due to differential solution activity along parallel to sub-parallel joints.
The lapie field may eventually turn into somewhat smooth limestone pavements.
• Depositional landforms:
o Stalactites: They hang as icicles from the roofs and are of different diameters. Normally they are
broad at their bases and taper towards the free ends.
o Stalagmite: They rise up from the floor of the caves. In fact, stalagmites form due to dripping water
from the surface or through the thin pipe, of the stalactite, immediately below it.
o Pillars: The stalagmite and stalactites eventually fuse to give rise to columns and pillars of
different diameters.
• Hence option (c) is the correct answer.
Q 17.A
• Luni is the largest river system of Rajasthan, west of Aravalli. It originates near Pushkar in two
branches, i.e. the Saraswati and the Sabarmati, which join with each other at Govindgarh. From here,
the river comes out of Aravalli and is known as Luni.
• It flows towards the west till Telwara and then takes a southwest direction to join the Rann of Kutch.
• During its total course of 495 km, Luni river passes through the states of Rajasthan and Gujarat.
• The entire river system is ephemeral.
• Mahi River is the only river in India that cuts the Tropic of Cancer twice, first in Madhya Pradesh from
where it flows towards Rajasthan and enters Gujarat where it cuts for the second time.
Q 18.D
• An eclipse season is one of only two periods during each year when eclipses can occur, due to the
variation in the orbital inclination of the Moon. Each season lasts about 35 days and repeats just short of
six months later, thus two full eclipse seasons always occur each year. Either two or three eclipses happen
each eclipse season. During the eclipse season, the inclination of the Moon's orbit is low, hence the Sun,
Moon, and Earth become aligned straight enough (in syzygy) for an eclipse to occur. In most calendar
years there are two lunar eclipses; in some years one or three or none occur. Solar eclipses occur
6 www.visionias.in ©Vision IAS
two to five times a year, five being exceptional; there last were five in 1935, and there will not be five
again until 2206. Hence statement 1 is not correct.
• If the Earth had a perfectly circular orbit centered around the Sun, and the Moon's orbit was also perfectly
circular and centered around the Earth, and both orbits were coplanar (on the same plane) with each other,
then two eclipses would happen every lunar month (29.53 days). A lunar eclipse would occur at every full
moon, a solar eclipse every new moon, and all solar eclipses would be the same type. Hence statement 2
is not correct.
Q 19.A
• The Western Ghats are locally known by different names such as Sahyadri in Maharashtra, Nilgiri hills
in Karnataka, and Tamil Nadu and Anaimalai hills and Cardamom hills in Kerala.
• The Western Ghats are comparatively higher in elevation and more continuous than the Eastern
Ghats.
• Their average elevation is about 1,500 m with the height increasing from north to south.
• ‘Anaimudi’ (2,695 m), the highest peak of Peninsular plateau is located on the Anaimalai Hills of the
Western Ghats followed by Dodabetta (2,637 m) on the Nilgiri hills.
• Most of the Peninsular rivers have their origin in the Western Ghats.
• Hence option (a) is the correct answer.
Q 20.A
• The islands of the Arabian sea include Lakshadweep and Minicoy.
The entire group of islands is broadly divided by the Eleventh-degree channel, north of which is the
Amini Island and to the south of the Cannanore Island.
• These are scattered between 8°N-12°N and 71°E -74°E longitude.
• These islands are located at a distance of 280 km-480 km off the Kerala coast.
• The entire island group is built of coral deposits.
• The entire group of islands is broadly divided by the Eleven-degree channel, north of which is the
Amini Island, and to the south of the Cannanore Island.
• The Islands of this archipelago have storm beaches consisting of unconsolidated pebbles,
shingles, cobbles, and boulders on the eastern seaboard.
• The maritime boundary between the Maldives and India (Lakshadweep) runs through the Eight-
degree channel, locally known as Addigiri Kandu.
• Saddle Peak or Saddle Hill is located on North Andaman Island in India's Andaman and the Nicobar
Islands.
• Narcondam is a small volcanic island located in the Andaman Sea. They are supposed to be dormant
volcanoes.
• Hence option (a) is the correct answer.

Q 21.D
• Statement 1 is not correct: A system of standard time is observed by all countries. Most countries adopt
their standard time from the central meridian of their countries. The Indian Government has accepted the
meridian of 82.5 east for the standard time which is 5 hours 30 minutes ahead of Greenwhich Meridian
Time. The whole world has in fact been divided into 24 Standard Time Zones, each of which differs from
the next by 15 longitudes or one hour in time.
• Statement 2 is not correct: Majority of the countries in the world have adopted their standard time from
the central meridian of their countries. There are some countries and regions that follow unusual time
7 www.visionias.in ©Vision IAS
zones. Many of them are in increments of a half-hour, different from the usual hour difference you
normally see. A great example of this is in India, where despite only being a half an hour different from
Coordinated Universal Time the time difference appears much larger. Locations that use non-standard
time zones include India, Sri Lanka, Afghanistan, Iran, Myanmar, Newfoundland, Regions of Australia,
Venezuela, Nepal, Chatham Islands, and the Marquesas Islands. Another example is the USA. Time in the
United States, by law, is divided into nine standard time zones covering the states, territories and
other US possessions, with most of the United States observing daylight saving time (DST) for
approximately the spring, summer, and fall months.
Q 22.A
• Himalayan mountain range in Asia separates the plains of the Indian subcontinent from the Tibetan
Plateau. They act as a climatic divide between the Indian sub-continent and Central Asia. They cast an
important influence on Indian climate by:
o protecting the subcontinent from the cold northern winds blowing from as far as Siberia. Hence,
statement 1 is correct.
o trapping the monsoon winds, forcing them to shed their moisture within the subcontinent. Hence,
statement 2 is correct.
• Differential heating of land and sea and the shift of ITCZ is responsible for creating different air pressure
zones causing a reversal in the direction of monsoon winds. Hence, statement 3 is not correct.
Q 23.A
• Karewas are the thick deposits of glacial clay and other materials embedded with moraines.
• To be more precise, Karewas are lacustrine deposits.
• According to geographers, the Karewa Formation is glacial- fluvial-lacustrine and aeolian loess of Plio-
Pleistocene age.
• The Kashmir Himalayas are also famous for Karewa formations, which are useful for the cultivation
of Zafran, a local variety of saffron.
• It lies between the Pir Panjal Range and the Great Himalayan Range, in Northwest India.
• Hence option (a) is the correct answer.
Q 24.A
• Recently it was reported that the yearly pokkali rice cultivation season looks to have taken a
beating with the COVID-19 lockdown preventing timely field preparations even as the monsoon
season appears to have set in.
• Pokkali is a unique rice variety in the coastal areas of Alappuzha, Ernakulam and Thrissur districts. The
salt water resistant variety grows tall, flourishes under flood-like conditions and grows entirely on
leftovers from half-a-year of fish culture in fields. The cycle is called ‘one fish and one rice’ in which fish
is grown up to April 15 after which fields are drained before the first bout of rains get them rid of salt. The
season goes up to October when they are brought under fish culture again.
• Porteresia is a form of wild rice that grows in saline estuaries in Bangladesh and India and is harvested
and eaten as a delicacy. The plant is salt-tolerant, and is seen as a possibly important source of salt-
tolerance genes for transfer to other rice species. Damodar is also a salt-tolerant variety of rice.
• The most famous rice variety introduced as a part of the Green Revolution in India was IR-8. It was
developed based on experience in developing the Norin variety of Japan and Ponlai variety of Taiwan. IR-
8 was short, stiff strawed, and highly responsive to the fertilizers.
Q 25.B
• Antecedent or Inconsequent Drainage: The Rivers that existed before the upheaval of the
Himalayas and cut their courses southward by making gorges in the mountains are known as the
antecedent rivers. The Indus, Satluj, Ganga, Sarju (Kali), Arun (a tributary of Kosi), Tista and
Brahmaputra are some of the important antecedent rivers, originating from beyond the Greater Himalayas.
Hence statement 1 is not correct.
• Consequent Rivers: The Rivers which follow the general direction of slope are known as the
consequent rivers. Most of the rivers of peninsular India are consequent rivers. For example, rivers like
Godavari, Krishna and Kaveri, descending from the Western Ghats and flowing into the Bay of Bengal,
are some of the consequent rivers of Peninsular India. Hence statement 2 is correct.
• Subsequent Rivers: A tributary stream that is eroded along an underlying belt of non-resistant rock
after the main drainage pattern (Consequent River) has been established is known as a subsequent
river. Due to the northward slope of the Peninsula towards the Great Plains, the rivers originating from
the Vindhyan and the Satpura ranges flow northward into the Ganga system. The Chambal, Sind, Ken,
Betwa, Tons and Son meet the Yamuna and the Ganga at right angles.
• Superimposed, Epigenetic (Discordant) or Superinduced Drainage: It is formed when a stream with
a course originally established on a cover of rock now removed by erosion, so that the stream or
8 www.visionias.in ©Vision IAS
drainage system is independent of the newly exposed rocks and structures. The Damodar, the
Subarnarekha, the Chambal, the Banas and the rivers flowing at the Rewa Plateau present some good
examples of superimposed drainage.
Q 26.A
• The structure of the earth is divided into three major components: the crust, the mantle, core (outer core
and inner core)
o Crust: It is the outermost solid part of the earth. It is brittle in nature. The thickness of the crust varies
under the oceanic and continental areas. Oceanic crust is thinner as compared to the continental crust.
The mean thickness of the oceanic crust is 5 km whereas that of the continental is around 30 km. The
continental crust is thicker in the areas of major mountain systems. It is as much as 70 km thick in the
Himalayan region.
o Mantle: The portion of the interior beyond the crust is called the mantle. The mantle extends from
Moho’s discontinuity to a depth of 2,900 km. The upper portion of the mantle is called the
asthenosphere. The word astheno means weak. It is considered to be extending up to 400 km. It is the
main source of magma that finds its way to the surface during volcanic eruptions. The crust and
the uppermost part of the mantle are called the lithosphere. Its thickness ranges from 10-200 km. The
lower mantle extends beyond the asthenosphere. It is in a solid-state. Hence, statement 2 is not
correct.
o Core: The core-mantle boundary is located at the depth of 2,900 km. The outer core is in a liquid state
while the inner core is in solid-state. The core is made up of very heavy material mostly constituted by
nickel and iron. It is sometimes referred to as the nife layer. Hence, statement 1 is correct.
Q 27.B
• Statement 1 is not correct: The uppermost layer over the earth’s surface is called the crust. It is
the thinnest of all the layers. It is about 35 km. on the continental masses and only 5 km. on the ocean
floors. The main mineral constituents of the continental mass are silica and alumina. It is thus called
sial (si-silica and al-alumina). The oceanic crust mainly consists of silica and magnesium; it is
therefore called sima (si-silica and ma-magnesium).
• Statement 2 is correct: The crust is composed of two basic rock types granite and basalt. The continental
crust is composed mostly of granite. The oceanic crust consists of a volcanic lava rock called basalt.
Basaltic rocks of the ocean plates are much denser and heavier than the granitic rock of the continental
plates.
Q 28.B
• There are three types of plate boundaries:
o Convergent Boundaries: Where the crust is destroyed as one plate dived under another. The
location where the sinking of a plate occurs is called a subduction zone. There are three ways in which
convergence can occur. These are: (i) between an oceanic and continental plate; (ii) between two
oceanic plates; and (iii) between two continental plates.
o Divergent Boundaries: Where a new crust is generated as the plates pull away from each other. The
sites where the plates move away from each other are called spreading sites. The best-known example
of divergent boundaries is the Mid-Atlantic Ridge. At this, the American Plate(s) is/are separated
from the Eurasian and African Plates.
o Transform Boundaries: Where the crust is neither produced nor destroyed as the plates slide
horizontally past each other. Transform faults are the planes of separation generally perpendicular to
the midoceanic ridges. As the eruptions do not take all along the entire crest at the same time, there is
a differential movement of a portion of the plate away from the axis of the earth. Also, the rotation of
the earth has its effect on the separated blocks of the plate portions
Q 29.D
• Peninsular Block of India: The northern boundary of the Peninsular Block may be taken as an irregular
line running from Kachchh along the western flank of the Aravali Range near Delhi and then roughly
parallel to the Yamuna and the Ganga as far as the Rajmahal Hills and the Ganga delta.
• Apart from these, the Karbi Anglong and the Meghalaya Plateau in the northeast and Rajasthan in
the west are also extensions of this block.
• The northeastern parts are separated by the Malda fault in West Bengal from the Chotanagpur
plateau. The Malda fault broadly separates the Meghalayan Plateau from the peninsular plateau.
Hence option (d) is the correct answer.

Q 30.A
• Winter season in India extends roughly from November to March. Winter Season is characterized by the
following weather phenomenon:
9 www.visionias.in ©Vision IAS
o Development of high pressure in the region lying to the north of the Himalayas. Hence, statement 1
is correct.
o This centre of high pressure gives rise to the flow of air at the low level from the north towards the
Indian subcontinent, south of the mountain range. The surface winds blowing out of the high pressure
centre over Central Asia reach India in the form of a dry continental air mass. Hence, statement 2 is
correct.
o All of Western and Central Asia remains under the influence of westerly jet streams winds along the
altitude of 9-13 km from west to east. Southern branch of the jet stream exercises an important
influence on the winter weather in India
o The western cyclonic disturbances enter the Indian subcontinent from the west and the northwest
during the winter months.
o Tropical cyclones originate over the Bay of Bengal and the Indian ocean.
o An easterly jet stream flows over the southern part of the Peninsula in Summer month of June. Hence,
statement 3 is not correct.
Q 31.D
• Central Highlands: also called the Madhya Bharat Pathar or Madhya Bharat Plateau.
• It is to the east of the Marwar or Mewar Upland and the north of the Narmada river. It covers the
major portion of the Malwa plateau.
• Most of the plateau comprises the basin of the Chambal river which flows in a rift valley. Hence
statement 2 is not correct.
• The Kali Sindh, flowing from Rana Pratap Sagar, The Banas flowing through Mewar plateau, and
The Parwan and the Parbati flowing from Madhya Pradesh are Chambal's main tributaries.
• It is a rolling plateau with rounded hills composed of sandstone. Thick forests grow here.
• To the north are the ravines or badlands of the Chambal river and Satpura ranges, to the south is the
Deccan plateau, and to the west are Aravalli ranges
• It slopes towards the north and northeastern directions. Hence statement 1 is not correct.

• Musi River or Musinuru is a tributary of the Krishna River in the Deccan Plateau flowing through
Telangana state in India. Hyderabad stands on the banks of Musi river,
Q 32.A
• Indus River System:
o It is one of the largest river basins of the world, covering an area of 11,65,000 sq. km (in India it
is 321, 289 sq. km and a total length of 2,880 km (in India 1,114 km). In Pakistan, the Indus river
basin covers around 520 000 km², or 65 percent of the territory, comprising the whole of the
provinces of Punjab and Khyber Pakhtunkhwa and most of the territory of Sindh province and the
eastern part of Balochistan. Hence statement 1 is correct.
o The basin area of Indus touches 4 countries (China, India, Afghanistan, Pakistan). Basin area is
distributed between Pakistan (47 percent), India (39 percent), China (8 percent) and
Afghanistan (6 percent). The Kabul River, the classical Cophes, is a 700-kilometer long river that
emerges in Maidan Wardak Province in the Sanglakh Range of the Hindu Kush mountains in
Afghanistan and is separated from the watershed of the Helmand River by the Unai Pass. The Kabul
River empties into the Indus River near Attock, Pakistan. Hence statement 2 is not correct.
10 www.visionias.in ©Vision IAS
o From its source in the Tibetan plateau, the Indus river initially flows northwest before bending south
to flow southwest through Pakistan to the Arabian sea.
Q 33.A
• Sirocco and Mistral are the local winds of the Mediterranean Climate.
o Sirocco: This is a hot, dry dusty wind which originates in the Sahara Desert. It may occur at any time
of the year but is most frequent in spring. It blows outwards in a southerly direction from the
desert interiors into the cooler Mediterranean Sea. It is known by different names in different
countries around Mediterranean.
o Mistral: It is cold wind from north, rushing down the Rhone valley in France over to the
Mediterranean sea.
• Foehn and Chinook are the local winds of the Temperate Continental Climate. The Foehn in
Switzerland and the Chinook on the eastern slopes of the Rockies have considerable effect on the local
pastures. They are hot winds and may raise the temperature by about 4 to 5 degree Celcius, thus melting
the snow in winter or early spring and is beneficial for agriculture.
• Warm moist air moves from the Mediterranean region and brings precipitation on the southern slopes of
the Alps. This air becomes warm due to adiabatic rise in temperature, loses its original characteristics
and descends on the northern slopes of the Alps in Switzerland, Germany and Austria. This dry warm air
is called as Foehn.
• Hence, the correct answer is option (a).
Q 34.B
• The geologic time scale (GTS) is a system of chronological dating that relates geological strata
(stratigraphy) to time. It is used by geologists, paleontologists, and other Earth scientists to describe the
timing and relationships of events that have occurred during Earth's history.
• Devonian Period, in geologic time, an interval of the Paleozoic Era that follows the Silurian Period and
precedes the Carboniferous Period, spanning between about 419.2 million and 358.9 million years
ago. The Devonian Period is sometimes called the “Age of Fishes” because of the diverse, abundant,
and, in some cases, bizarre types of these creatures that swam Devonian seas. Forests and the coiled
shell-bearing marine organisms known as ammonites first appeared early in the Devonian. Late in the
period the first four-legged amphibians appeared, indicating the colonization of land by vertebrates.

11 www.visionias.in ©Vision IAS


Q 35.B
• The Raja Parba of Odisha is one of the most popular festivals of the state. Dedicated to Bhuma Devi,
the consort of Lord Vishnu, Raja Parba celebrates womanhood. It happens in the month of June.
o The first day of the festival is called Pahili Raja, the second is Mithuna Sankranti and the third Bhu
daha or Basi Raja. The preparation begins one day before Pahili Raja, and it is called Sajabaja.
Primarily, it is a time for the unmarried girls to prepare for their matrimony. They follow various
customs related to the festival by consuming nutritious food like Podapitha, not walking barefoot,
taking a bath on the first day, and merrily swinging on ropes attached to a tree.
o During the Parba, Odia people do no undertake any construction works or tilling that requires the
earth to be dug. And by not doing such activities, they pay ode to Mother Earth who needs a break
from routine work.
o The festival concludes with a custom called Vasumati Snana or the bathing of Bhuma Devi. Women
worship a stone that symbolises the Mother Earth. They give her a bath with turmeric paste and offer
her flowers and smear her with Sindoor.
o This festival is also associated with the end of the summer season and the arrival of the monsoon. And
therefore, it is also associated with agriculture and cultivation related communities and activities.
• Thrissur Pooram is an annual Hindu festival held in Kerala. It is celebrated on the day when the star
sign “Pooram” occurs in the Malayalam month of “medam (April-May)”. The festival was the brainchild
of Raja Rama Varma, famously known as Sakthan Thampuran, the Maharaja of Cochin (1790–1805).
o Pooram officially begins with Kodiyettam (flag hoisting ceremony) in which all the participating
temples of the festival are present.
o The Pooram consists of ten temples in and around Thrissur and is considered to be a ceremony where
these deities come together to pay obeisance to Lord Shiva at the Vadakkunnathan Temple, located in
the centre of the town.
o One of the hallmarks of the festival is the percussion ensemble consisting of traditional instruments
like chenda, maddalam, edakka, thimila and kombu.
o The seventh day of the pooram is the last day. It is also known as “Pakal Pooram”.
o Pooram, though a Hindu ritual, has grown to encompass all religious and cultural strains of Kerala.
Both the Muslim and Christian communities participate in the festival in a variety of ways
highlighting secular credentials of the state.
o It was cancelled for the first time amid COVID-19 for the first time since its inception.
• Ambubachi Mela or festival marks the annual menstruation of the presiding Goddess in the Kamakhya
Temple, Guwahati, Assam. The temple’s sanctum sanctorum houses the yoni (female
genital) symbolised by a rock.
o Kamakhya is one of 51 shakti peethas or holy sites for the followers of the Shakti cult, each
representing a body part of the Sati, Lord Shiva’s companion.
o The temple is situated on the Nilachal Hills, whose northern face slopes down to the Brahmaputra
river.According to the legends, it was built by the demon king Narakasura but recordsare
available only from 1565 when Koch king Naranarayana rebuilt the temple.
o It was cancelled recently amid COVID-19.
Q 36.C
• Narmada, Tapi, Mahi and Damodar are the rives flow in a rift valley. Hence option c is correct.
• Narmada flows westwards in a rift valley between the Satpura and Vindhya ranges. Damodar River flows
through rift valley in Chhota Nagpur Plateau. The Tapti and Mahi River also flow through rift valleys, but
between different ranges.

Q 37.D
• Rising from the height of 150 m above the river plains up to an elevation of 600-900 m is the irregular
triangle known as the Peninsular plateau.
• Delhi ridge in the northwest, (extension of Aravalis), the Rajmahal Hills in the east, the Gir range in
the west, and the Cardamom Hills in the south constitute the outer extent of the Peninsular plateau.
• However, an extension of this is also seen in the northeast, in the form of Shillong and Karbi-
Anglong plateau. Hence statement 3 is correct.
• This is one of the oldest and the most stable landmass of India. But, the Bhima fault needs special
mention, because of its recurrent seismic activities.
• This Peninsular plateau has undergone recurrent phases of upliftment and submergence accompanied by
crustal faulting and fractures. Hence statement 2 is not correct.
• The general elevation of the plateau is from the west to the east, which is also proved by the pattern of
the flow of rivers. Hence statement 1 is correct.
12 www.visionias.in ©Vision IAS
• The western and northwestern part of the plateau has an emphatic presence of black soil.
• On the basis of the prominent relief features, the Peninsular plateau can be divided into three broad
groups: (i) The Deccan Plateau(ii) The Central Highlands(iii) The Northeastern Plateau.

Q 38.D
• The Government of India adopted the nation-wide Forest Conservation Policy in 1952, which was
further modified in 1988.
• The forest policy aimed at:
o Bringing 33% of the geographical areas under forest cover, through social forestry and
afforestation on degraded land.
o Maintaining environmental stability and to restore forests where ecological balance was disturbed.
o Conserving the natural heritage of the country, its biological diversity and genetic pool.
o Checks soil erosion, extension of the desert lands and reduction of floods and droughts.
o Increasing the productivity of forests to make timber, fuel, fodder and food available to rural
population dependent on forests, and encourage the substitution of wood.
o Creating of a massive peoples movement involving women to encourage planting of trees, stop
felling of trees and thus, reduce pressure on the existing forest. Hence, all the statements are
correct.
• The government has also enacted the Forest Conservation Act in 1980 and has setup a National Forest
Commission in 2003, for assessment and integrated development of forests in India.

Q 39.D
• Statement 1 is not correct: Many planets in our solar system have more than one moon. Mars has two
moons, Jupiter has 67, Saturn 62, Uranus 27, Neptune 14. Those numbers keep changing and updating. Of
the inner planets, Mercury and Venus have no natural satellites; Earth has one large natural satellite,
known as the Moon; and Mars has two tiny natural satellites, Phobos and Deimos.
• Statement 2 is not correct: Venus is one of just two planets that rotate from east to west. Only Venus
and Uranus have this "backwards" rotation. It completes one rotation in 243 Earth days — the longest day
of any planet in our solar system, even longer than a whole year on Venus. Uranus orbits the sun quite
unusually, being the only planet whose equator is nearly at a right angle to its orbit, with a tilt of 97.77
degrees. Because of this, it rotates in the opposite direction than most planets, from East to West.
• Statement 3 is not correct: A ring system around a planet is also known as a planetary ring system. The
most prominent and most famous planetary rings in the Solar System are those around Saturn, but the
other three giant planets (Jupiter, Uranus, and Neptune) also have ring systems. Hence, all the statements
are not correct.
Q 40.C
• Recently the Food Safety and Standards Authority of India (FSSAI) released the State Food Safety
Index in which Gujarat, Tamil Nadu and Maharashtra have topped. It is an index that ranks states ensuring
food safety in 2019-20. The index ranks states on five parameters of food safety: human resources
and institutional data, compliance, food testing facility, training and capacity building besides
consumer empowerment. Hence both statements 1 and 2 are correct.
• Gujarat, Tamil Nadu and Maharashtra have topped in the list of larger states on the index. Among the
smaller, Goa came first followed by Manipur and Meghalaya. Among Union Territories, Chandigarh,
Delhi and Andaman Islands secured top ranks.
• This is the second index on food safety, which FSSAI released on the occasion of World Food Safety Day
with the theme "Food Safety is everyone's business" that was dedicated to those in the supply chain who
have ensured the uninterrupted availability of safe food during this COVID-19 pandemic.
Q 41.A
• An earthquake in simple words is shaking of the earth. It is a natural event. It is caused due to the release
of energy, which generates waves that travel in all directions.
• Statement 1 is correct: All natural earthquakes take place in the lithosphere. The release of energy
occurs along a fault. A fault is a sharp break in the crustal rocks. Rocks along a fault tend to move in
opposite directions. As the overlying rock strata press them, the friction locks them together. However,
their tendency to move apart at some point of time overcomes the friction. As a result, the blocks get
deformed and eventually, they slide past one another abruptly. This causes a release of energy, and the
energy waves travel in all directions.
• Statement 2 is not correct: The point where the energy is released is called the focus of an earthquake,
alternatively, it is called the hypocentre. The energy waves travelling in different directions reach the
surface.
13 www.visionias.in ©Vision IAS
• The point on the surface, nearest to the focus, is called epicentre. It is the first one to experience the
waves. It is a point directly above the focus.
• Lithosphere
o It is the rocky outer part of the Earth. It is made up of the brittle crust and the top part of the upper
mantle. The lithosphere is the coolest and most rigid part of the Earth

Q 42.C
• PM Street Vendor’s AtmaNirbhar Nidhi (PM SVANidhi) is a special micro-credit facility for street
vendors launched recently by the Ministry of Housing and Urban Affairs. It is a Central Sector
Scheme to facilitate street vendors to access affordable working capital loan for resuming their livelihoods
activities, after easing of lockdown. Hence statement 3 is not correct.
• Objectives of the Scheme:
o To facilitate working capital loan up to Rs. 10,000 at subsidized rate of interest; Hence
statement 1 is not correct.
o To incentivize regular repayment of loan; and
o To reward digital transactions.
• Salient Features of the scheme:
o Interest subsidy on timely/ early repayment @ 7%
o Monthly cash-back incentive on digital transactions
o Higher loan eligibility on timely repayment of the first loan.
o It covers street vendors/ hawkers vending in urban areas, as on or before March 24, 2020, including
the vendors of surrounding peri-urban and rural areas.
• Lending institutions that will provide credit under it: Scheduled Commercial Banks, Regional Rural
Banks, Small Finance Banks, Cooperative Banks, Non-Banking Financial Companies, Micro-Finance
Institutions and SHG Banks.
• The Scheme shall be implemented up to March, 2022.
• KYC documents required (either of them): Aadhaar Card, Voter’s Identity Card, Driving Licence,
MNREGA Card, PAN Card.
• No collateral security is required to avail this loan. Hence statement 2 is correct.
• If a loanee name is in the list of surveyed vendors, but he does not have either Identity Card or Certificate
of Vending then also he is can avail of this scheme. A Provisional Certificate of Vending would be issued
to vendors through an IT-based Platform. The Banking Correspondent/ Agent will help in filling up the
application and upload the documents in a mobile App/ Portal.

Q 43.B
• A tectonic plate (also called lithospheric plate) is a massive, irregularly-shaped slab of solid rock,
generally composed of both continental and oceanic lithosphere. Plates move horizontally over the
asthenosphere as rigid units.
• A plate may be referred to as the continental plate or oceanic plate depending on which of the two occupy
a larger portion of the plate. Pacific plate is largely an oceanic plate whereas the Eurasian plate may be
called a continental plate. The theory of plate tectonics proposes that the earth’s lithosphere is divided into
seven major and some minor plates
• The major plates are as follows:
o Antarctica and the surrounding oceanic plate
o North American Plate (with western Atlantic floor separated from the South American plate along the
Caribbean islands)
o South American Plate (with western Atlantic floor separated from the North American plate along
the Caribbean islands)
o Pacific plate
o India-Australia-New Zealand plate
o Africa with the eastern Atlantic floor plate
o Eurasia and the adjacent oceanic plate.
• Some important minor plates are listed below:
o Cocos plate: Between Central America and Pacific plate
o Nazca plate: Between South America and Pacific plate
o Arabian plate: Mostly the Saudi Arabian landmass
o Philippine Plate: Between the Asiatic and Pacific plate

14 www.visionias.in ©Vision IAS


Q 44.C
• The Dead Sea is landlocked. All the minerals of the surrounding countryside get washed into one pool,
which in turn gets baked by the sun. This makes water in Dead sea very high in salt concentration.
• The concentration of salt reaches 34%. The extremely high concentration of dissolved mineral salts in
the water causes the water to be denser than that of plain fresh water. Since our body weight is lighter
(less dense) than the density of the water, our body is more buoyant in the Dead Sea, making it easy to
float. Hence option (c) is the correct answer.

Q 45.D
• In the mountainous areas, the decrease in temperature with the increase in altitude leads to the formation
of Montane forests. The Montane forests consists of two types of vegetation:
o The wet temperate type of forests are found between the height of 1000 and 2000 meters,
with evergreen broad-leaf trees such as oaks and chestnuts. Hence, statement 1 is correct.
o The temperate forests between the height of 1500 and 3000 meters, with coniferous trees like pine,
deodar, silver fir, spruce and cedar.
• The common animals found in these forests are Kashmir stag, spotted dear, wild sheep, jack rabbit,
Tibetan antilope, yak, snow leopard, squirrels, shaggy horn wild ibex, bear and rare red panda, sheep and
goats with thick hair. Hence, statement 2 is correct.
• These forests cover mostly the southern slopes of the Himalayas, places having high altitude in
southern portion of the Western Ghats range in Karnataka, Kerala and Tamil Nadu, at elevations
over 1000 meters and north-east India. Hence, statement 3 is correct.

Q 46.D
• High temperatures and abundant rainfall in the hot and wet equatorial regions support a mix of
luxuriant type of vegetation -the tropical rain forest.
• The growing season here is all the year round and hence the forests are evergreen.
• Amazon tropical rain forests, for its lush green and broad leaves, are known as Selvas.
• The region's vegetation comprises a multitude of evergreen trees that yield tropical hardwood,
e.g. mahogany, ebony, greenheart, cabinet wood.
• There are also small palm trees and climbing plants like Lianas or Rattans, which are epiphytic (A
plant that grows on another plant and depends on it for support but not food).
• Under the trees, they also grow a wide variety of ferns, orchids, and Lalang.
• This leads to a distinct layer arrangement in the forests, with tall trees and their dense canopy at top,
smaller trees forming the next layer and then the ground is covered with herbaceous ferns and shrubs.
• Trees of single species are very scarce in such vegetation. and hence commercial exploitation of the
tropical timber is most difficult.
• Hence option (d) is the correct answer.
15 www.visionias.in ©Vision IAS
Q 47.A
• Statement 1 is correct: Extra-tropical cyclone, also known as wave cyclone or mid-latitutde cyclone, is a
type of storm system formed in middle or high latitudes, in regions of large horizontal temperature
variations called frontal zones. They develop when a wave forms on a frontal surface separating a warm
air-mass from a cold air mass. Thus, extra-tropical cyclones form along the polar front. In the northern
hemisphere, warm air blows from the south and cold air from the north of the front. When the pressure
drops along the front, the warm air moves northwards and the cold air move towards the south setting in
motion an anti-clockwise cyclonic circulation. The cyclonic circulation leads to a well-developed extra-
tropical cyclone, with a warm front and a cold front.
• Statement 2 is correct and statement 3 is not correct: They cover a larger area and can originate over
the land and sea. The wind velocity in a tropical cyclone is much higher and it is more destructive. The
extra-tropical cyclones move from west to east in the northern hemisphere.

Q 48.C
• Minor relief features like atoll, seamount, guyot, Mid-Oceanic Ridges etc predominate in different parts of
the ocean floor.
• Mid-Oceanic Ridges (MOR)
o A mid-oceanic ridge is an underwater mountain range, formed by plate tectonics. This uplifting of
the ocean floor occurs when convection currents rise in the mantle beneath the oceanic crust
and create magma where two tectonic plates meet at a divergent boundary. This forms an
interconnected chain of mountain system within the ocean. It is the longest mountain-chain on the
surface of the earth though submerged under the oceanic waters.
• Seamount
o It is a mountain with pointed summits, rising from the seafloor that does not reach the surface of the
ocean. Seamounts are volcanic in origin. These can be 3,000-4,500 m tall. The Emperor seamount,
an extension of the Hawaiian Islands in the Pacific Ocean, is a good example.
• Submarine canyons
o Submarine canyons are defined as steep-walled, sinuous valleys with V-shaped cross sections, axes
sloping outward as continuously as river-cut land canyons and relief comparable to even the largest of
land canyons. Submarine canyons are formed by the repeated erosion of the slope by turbidity
currents flowing down the canyon axis. The Hudson Canyon is the best known canyon in the world.
• Atoll
o These are low islands found in the tropical oceans consisting of coral reefs surrounding a central
depression. It may be a part of the sea (lagoon), or sometimes form enclosing a body of fresh,
brackish, or highly saline water.
• Hence only MOR and Seamounts are volcanic in origin.

Q 49.C
• Dooars or Duars are the alluvial floodplains in northeastern India that lie south of the outer foothills
of the Himalayas and north of the Brahmaputra River basin. Hence statement 1 is correct.
• This region is about 30 km wide and stretches over about 350 km from the Teesta River in West Bengal
to the Dhanshiri River in Assam.
• The region forms the gateway to Bhutan. It is part of the Terai-Duar savanna and grasslands ecoregion.
Duar region is rich in wildlife.
• The British, taking advantage of the physical conditions such as moderate slope, thick soil cover with
high organic content, well-distributed rainfall throughout the year, and mild winters, introduced tea
plantations in this region. Darjeeling and Assam tea gardens are famous world over for the quality of
their tea leaf. Hence statement 2 is correct.
16 www.visionias.in ©Vision IAS
Q 50.C
• Statement 1 is correct: Twilight is the time between day and night when there is light outside, but
the Sun is below the horizon. This is caused by the fact that during periods of dawn and twilight the
earth receives diffused or refracted light from the Sun. Twilight occurs when Earth's upper atmosphere
scatters and reflects sunlight which illuminates the lower atmosphere. Astronomers define the three stages
of twilight – civil, nautical, and astronomical – on the basis of the Sun's elevation which is the angle that
the geometric center of the Sun makes with the horizon.
• Statement 2 is correct: The length of twilight depends on latitude. Equatorial and tropical regions tend
to have shorter twilights than locations on higher latitudes. During summer months at higher latitudes,
there may be no distinction between astronomical twilight after sunset and astronomical twilight before
sunrise. This happens when the angle the Sun makes with the horizon – also known as the Solar Elevation
Angle – is less than 18 degrees during the local midnight. Similarly, higher latitudes may experience an
extended period of nautical twilight – if the Sun remains less than 12 degrees below the horizon
throughout the night.
Q 51.B
• The pattern of flow of water in a river channel over a year is known as its regime.
• The north Indian rivers originating from the Himalayas are perennial as they are fed by glaciers through
snow melt and also receive rainfall water during rainy season.
• The rivers of South India do not originate from glaciers and their flow pattern witnesses fluctuations. The
flow increases considerably during monsoon rains. Thus, the regime of the rivers of South India is
controlled by rainfall which also varies from one part of the Peninsular plateau to the other.
Q 52.D
• Recently, the colour of water in Maharashtra’s Lonar lake, formed after a meteorite hit the Earth some
50,000 years ago, has changed to pink with experts attributing it to the salinity and presence of algae in
the water body. Located around 500 km from Mumbai, the Lonar lake in Buldhana district is a popular
tourist hub and also attracts scientists from all over the world. Of late, the change in colour of water of the
lake, having a mean diameter of 1.2 km, has not only surprised locals, but also nature enthusiasts and
scientists.
• The lake, which is a notified national geo-heritage monument, has saline water with pH of 10.5. There
are algae in the water body. The salinity and algae can be responsible for this change. There is no
oxygen below one meter of the lake’s water surface. There is an example of a lake in Iran, where water
becomes reddish due to increase in salinity. The low level of water may lead to increased salinity and
change in the behaviour of algae because of atmospheric changes and this may be the reason for
colour change.
• In case of a natural phenomenon, there are fungi which generally give a greenish colour to water most of
the times. This (the current colour change) seems to be a biological change in the Lonar crater.
Q 53.B
• Recently, U.S. President Donald Trump invited Prime Minister Narendra Modi to the next G-7
summit to be hosted by the U.S. The Group of Seven (G-7) is a forum of the seven countries with the
world's largest developed economies—France, Germany, Italy, Japan, the United States, the United
Kingdom, and Canada—whose government leaders meet annually on international economic and
monetary issues.
• The member country holding the G7 presidency is responsible for organizing and hosting the year's
summit. As of 2018, the seven countries involved represent 58% of the global net wealth ($317 trillion)
and more than 46% of the global gross domestic product (GDP) based on nominal values, and more than
32% of the global GDP based on purchasing power parity. The European Union is an invitee to G7.
Q 54.D
• Ocean currents are like river flow in oceans. They represent a regular volume of water in a definite path
and direction. Ocean currents are influenced by two types of forces namely
o primary forces that initiate the movement of water
o secondary forces that influence the currents to flow.
• The primary forces that influence the currents are:
o heating by solar energy;
o wind;
o gravity;
o coriolis force.
• Heating by solar energy causes the water to expand. That is why, near the equator the ocean water is
about 8 cm higher in level than in the middle latitudes. This causes a very slight gradient and water tends
to flow down the slope.
17 www.visionias.in ©Vision IAS
• Wind blowing on the surface of the ocean pushes the water to move. Friction between the wind and the
water surface affects the movement of the water body in its course.
• Gravity tends to pull the water down to pile and create gradient variation. The Coriolis force intervenes
and causes the water to move to the right in the northern hemisphere and to the left in the southern
hemisphere. These large accumulations of water and the flow around them are called Gyres. These
produce large circular currents in all the ocean basins.
• Differences in water density affect vertical mobility of ocean currents. Water with high salinity is denser
than water with low salinity and in the same way cold water is denser than warm water. Denser water
tends to sink, while relatively lighter water tends to rise.
• Cold-water ocean currents occur when the cold water at the poles sinks and slowly moves towards the
equator. Warm-water currents travel out from the equator along the surface, flowing towards the poles to
replace the sinking cold water.
• The topography and shape of ocean basins and nearby landmasses also influence ocean currents.
• Hence option (d) is the correct answer.
Q 55.A
• Statement 2 is not correct: The maximum temperature of the oceans is always at their surfaces because
they directly receive the heat from the sun and the heat is transmitted to the lower sections of the oceans
through the process of conduction. It results into decrease of temperature with the increasing depth,
but the rate of decrease is not uniform throughout. Temperature structure of oceans can be described
as three layer system.
o First layer (Top layer) is about 500m thick with temperatures ranging between 20 to 25oc.
o Second layer (Thermocline) is characterised by rapid decrease in temperature with increasing depth.
It is about 500m-1000m thick.
o Third layer is very cold and extends upto deep ocean floor. Here temperature change with the depth
is very slight.
• Statement 1 is not correct and Statement 3 is correct: The average temperature of surface water of the
oceans is about 27°C and it gradually decreases from the equator towards the poles. The oceans in the
northern hemisphere record relatively higher temperature than in the southern hemisphere. The
highest temperature is not recorded at the equator but slightly towards north of it. The average annual
temperatures for the northern and southern hemisphere are around 19° C and 16° C respectively. This
variation is due to the unequal distribution of land and water in the northern and southern hemispheres.
• Hence option (a) is the correct answer.
Q 56.C
• The latitudinal and longitudinal extent of India is roughly 30 degrees, whereas the actual distance
measured from north to south extremity is 3,214 km, and that from east to west is only 2,933
km. Hence both statements 1 and 2 are correct.
• From the values of longitude, it is quite discernible that there is a variation of nearly 30 degrees, which
causes a time difference of nearly two hours between the easternmost and the westernmost parts of our
country.
• India is away from the equator. So, the lines of longitude come nearer and the East-West distance
becomes less than the North-South distance.
• The distance between any two lines of longitude is maximum at the equator and goes on decreasing as one
goes from the equator to the poles. The distance between lines of latitude remains the same at all
latitudes.
Q 57.B
• The oceanic circulation transports heat from one latitude belt to another in a manner similar to the
heat transported by the general circulation of the atmosphere. The cold waters of the Arctic and Antarctic
circles move towards warmer water in tropical and equatorial regions, while the warm waters of the lower
latitudes move polewards.
• The Coriolis force causes the water to move to the right in the northern hemisphere and to the left in
the southern hemisphere. Hence statement 2 is correct.
• West coasts of the continents in tropical and subtropical latitudes (except close to the equator) are
bordered by cool waters. Their average temperatures are relatively low with a narrow diurnal and annual
ranges. There is fog, but generally, the areas are arid. West coasts of the continents in the middle and
higher latitudes are bordered by warm waters which cause a distinct marine climate. They are
characterized by cool summers and relatively mild winters with a narrow annual range of
temperatures. Hence statement 1 is not correct.
• Warm currents flow parallel to the east coasts of the continents in tropical and subtropical latitudes. This
results in warm and rainy climates.
18 www.visionias.in ©Vision IAS
• Currents are referred to by their “drift”. Usually, the currents are strongest near the surface and may attain
speeds over five knots. The strength of a current refers to the speed of the current. A fast current is
considered strong. Current is usually strongest at the surface and decreases in strength (speed) with
depth.
• Hence option (d) is the correct answer.
Q 58.A
• Statements 1 and 2 are correct: Indian Gas Exchange (IGX), the first nationwide online delivery-
based gas trading platform was recently launched by the Minister of Petroleum and Natural Gas. It
is expected to lead to market-driven pricing and boost consumption by improving availability. IGX
is fully automated with a web-based interface to provide seamless trading experience to the customers and
is powered by best-in-class technology from GMEX, one of the world’s leading digital exchange trading
and post-trade technology providers.
• Statement 3 is not correct: The IGX platform is incorporated by the Indian Energy Exchange
(IEX), India’s leading energy market platform. Incorporated as a wholly-owned subsidiary of the IEX,
IGX will enable market participants to trade in standardized gas contracts.
• IGX will play an instrumental role in transforming India’s gas markets, positioning India as a sustainable
economy, and enhancing the industry’s competitiveness. The competitive price discovery will facilitate
the availability of gas at lower prices for cross-spectrum of industries across India, stimulate demand, and
facilitate greater investments in domestic gas exploration.
Q 59.B
• Recent Context: India secured the168 rank in the 12th edition of the biennial Environment Performance
Index (EPI Index 2020) — that measured the environmental performance of 180 countries — and was
released by the Yale University in June 2020. India’s rank was 177 (with a score of 30.57 out of 100) in
2018. The country scored 27.6 out of 100 in the 2020 index. The global index considered 32 indicators of
environmental performance, giving a snapshot of the 10-year trends in environmental performance at the
national and global levels. Overall EPI rankings indicate which countries are best addressing the
environmental challenges that every nation faces.
• This index has been developed by Yale University and Columbia University in collaboration with
the World Economic Forum and the Joint Research Centre of the European Commission.
Q 60.D
• Mizoram is known as the land of rolling mountains as it has a huge number of mountains, the formation of
most mountains is accompanied by the formation of foreland basin or in simple terms valley type
depression which runs parallel to mountains. These depression get accumulated with unconsolidated
deposits known as molasses basin. hence it is also known as the molasses basin of India.
Q 61.C
• The China Type of climate is also called as the temperate monsoon climate. The great landmass of the
Asiatic continent with its mountainous interior induces great pressure changes between summer and
winter.
• Intense heating in 'the heart of Asia' sets up a region of low pressure in summer that draws in the
tropical Pacific air stream as the rain-bearing South-East Monsoon. Hence statement 1 is correct.
• In winter, a steep pressure gradient is set up between the cold interiors of Mongolia and Siberia,
and the warmer Pacific coastlands. The continental polar air stream flows outwards as the North-West
Monsoon. There is little rain but considerable snow on the windward slopes of certain regions. Hence
statement 2 is correct.
Q 62.D
• Social forestry means the management and protection of forests and afforestation on barren lands with the
purpose of helping in the environmental, social and rural development.
• The National Commission on Agriculture (1976) has classified social forestry into three categories. These
are Urban forestry, Rural forestry and Farm forestry:
o Urban forestry pertains to the raising and management of trees on public and privately owned lands in
and around urban centres such as green belts, parks, roadside avenues, industrial and commercial green
belts, etc.
o Rural forestry lays emphasis on promotion of agro-forestry and community-forestry.
✓ Agro-forestry is the raising of trees and agriculture crops on the same land inclusive of the waste
patches. Hence option (d) is the correct answer.
✓ Community forestry involves the raising of trees on public or community land such as the village
pasture and temple land, roadside, canal bank, strips along railway lines, and schools, etc. It aims at
providing benefits to the community as a whole.
• Farm forestry means the process under which farmers grow trees for commercial and non-commercial
purposes on their farmlands.
19 www.visionias.in ©Vision IAS
Q 63.C
• Drumlins are depositional landforms of glaciation. These are smooth oval-shaped ridge-like features
composed mainly of glacial till with some masses of gravel and sand. The long axes of drumlins are
parallel to the direction of ice movement. One end of the drumlins facing the glacier called the
stoss end is blunter and steeper than the other end called the tail. The stoss end gets blunted due to
pushing by moving ice. Drumlins give an indication of the direction of the glacier movement.
• Serrated ridges are erosional landforms formed by glacial action.
• Moraines are long ridges of deposits of glacial till. Unlike drumlins, they do not indicate the direction of
glacial movement.
• Eskers are a long, winding ridge of stratified sand and gravel and very coarse materials like
boulders and blocks.
• Hence option (c) is the correct answer.

Q 64.C
• The Western Ghats in the west, and the Eastern Ghats in the east border the Deccan Plateau from both the
sides.
• The Western Ghats are comparatively higher in elevation. Hence statement 1 is correct.
• They are also more continuous than the Eastern Ghats. Hence statement 2 is not correct.
• Their average elevation is about 1,500 m with the height increasing from north to south. Hence,
statement 3 is correct.
• ‘Anaimudi’ (2,695 m), the highest peak of Peninsular plateau is located on the Anaimalai Hills of
the Western Ghats.
• Most of the Peninsular rivers have their origin in the Western Ghats.
• The Eastern Ghats comprising the discontinuous and low hills are highly eroded by rivers such as the
Mahanadi, the Godavari, the Krishna, the Kaveri, etc.
• Some of the important ranges include the Javadi hills, the Palkonda range, the Nallamala hills, the
Mahendragiri hills, etc.
• The Eastern and the Western Ghats meet each other at the Nilgiri hills.

Q 65.B
• Indian coastal plains can be broadly divided into two: (i) the western coastal plains; (ii) the eastern coastal
plains.
• Western Coastal plains:
o an example of submerged coastal plains.
o Because of this submergence, it is a narrow belt and provides natural conditions for the development
of ports and harbor.
o Extending from the Gujarat coast in the north to the Kerala coast in the south i.e., in five Indian
states ((Maharashtra, Goa, Karnataka Kerala, and Gujarat). Hence statement 1 is correct.
o They are narrow in the middle and get broader towards the north and south. Hence statement 2
is not correct.
o The rivers flowing through this coastal plain do not form any delta.
o The Malabar coast has got certain distinguishing features in the form of ‘Kayals’ (backwaters),
which are used for fishing, inland navigation, and also due to its special attraction for tourists. Hence,
statement 3 is correct.
• Eastern Coastal plains:
o It is broader and is an example of an emergent coast.
o There are well-developed deltas here, formed by the rivers flowing eastward into the Bay of Bengal
o Because of its emergent nature, it has less number of ports and harbors.
Q 66.B
• Recently, India was elected as a non-permanent member of the United Nations Security Council
with an overwhelming majority. The two-year term will begin on 1 January 2021, and along with India —
Ireland, Mexico and Norway were also elected to the council.
• India was the sole candidate from the Asia-Pacific region vying for a non-permanent membership. Its
candidature was unanimously endorsed by the 55-member Asia-Pacific Group in June last year. This is
the eighth time India has been elected a non-permanent member of the UNSC. Previously, it has been
elected as a member for the period of 1950-1951, 1967-1968, 1972-1973, 1977-1978, 1984-1985, 1991-
1992 and 2011-2012.
• The UNSC consists of 15 members — five permanent members and 10 non-permanent members. The
five permanent members are the US, UK, Russia, China and France. Every year, five non-permanent
20 www.visionias.in ©Vision IAS
members are elected for a tenure of two years. To be elected as a non-permanent member of the council,
each member-country requires a two-third majority of the entire assembly.
• At the eighteenth session of the UNSC in 1963, the General Assembly had decided that the non-
permanent members of the council should be elected according to the following pattern: Five members
from African and Asian countries, one from Eastern Europe, two Latin American countries and two from
Western Europe and other countries. The current non-permanent members are Belgium, Dominican
Republic, Estonia, Germany, Indonesia, Niger, Saint Vincent and the Grenadines, South Africa and
Tunisia.
Q 67.B
• Variability of Rainfall refers to the degree to which rainfall amounts vary across an area or through time.
There are two types (or components) of rainfall variability:
o Areal: The variation of rainfall amounts at various locations across a region for a specific time
interval. (Time does not vary.) Example: Annual average precipitation
o Temporal: The variation of rainfall amounts at a given location across a time interval. (Area does not
vary.)
• The variability of rainfall is computed with the help of the following formula:
o C.V. = (Standard Deviation/Mean)*100 ......(C.V. is the coefficient of variation)
o The values of the coefficient of variation show variability of rainfall in India.
• Different regions in India experience different ranges of variability:
o Variability less than 25 percent:
✓ Western coasts, Western Ghats, northeastern Peninsula, eastern plains of the Ganga, northeastern
India, Uttarakhand and Himachal Pradesh and south-western part of Jammu and Kashmir. These
areas have an annual rainfall of over 100 cm.
o Variability greater than 50 percent:
✓ Western part of Rajasthan, northern part of Jammu and Kashmir and interior parts of the
Deccan plateau. These areas have an annual rainfall of less than 50 cm.
o Variability of 25-50 percent:
✓ Rest of India such as Western Gangetic Plains.
✓ These areas receive an annual rainfall between 50 -100 cm

21 www.visionias.in ©Vision IAS


• Pacific Ocean Currents includes various cold and warm current which moves clockwise circulation
in Northern Pacific Ocean and Anticlockwise circulation in South Pacific Ocean thereby influencing the
climatic pattern in the coastal regions. The currents are mentioned below:
o North Equatorial Current (Warm)
o South Equatorial Current (Warm)
o Counter Equatorial Current (Warm)
o Kuroshio System (Warm)
o North Pacific Drift (Warm)
o Oyashio Current (Cold)
o California Current (Cold)
o Peruvian or Humboldt Current (Cold)
o East Australia Current (Warm)
o Alaska current (warm)

• Hence option (a) is the correct answer.

Q 76.D
• Since tides are caused by the earth-moon-sun positions which are known accurately, the tides can be
predicted well in advance. This helps the navigators and fishermen plan their activities. Tidal flows are of
great importance in navigation.
• Importance of tides:
• Fish may concentrate during ebb tides. Commercial fishermen follow the tides and learn to fish during
levels of highest concentration to improve their economic investment and to make more efficient use of
their time.
• Tides affect other aspects of oceanic life, including the reproductive activities of fish and ocean plants.
Floating plants and animals ride the tidal currents between the breeding areas and deeper waters. The
tides help remove pollutants and circulate nutrients ocean plants and animals need to survive.

24 www.visionias.in ©Vision IAS


• Crabs, mussels, snails, seaweed and other edible sea life inhabit the tidal zone. Small tide pools may also
contain small fish and sea vegetables. The sea life found in these regions are often harvested for food.
Without the regular washing of the tides, these complex and abundant creatures would die and food
resources would diminish.
• Tides affect the depth and currents in and around coastal areas. Ships may need to navigate the
waters during high tide in some areas or risk running aground. Pilots take into consideration the water
level, width of channels and direction of the water flow to determine the best time to travel.
• Two high tides and two low tides occur during every 24-hour period. The predictability of the tides, fast
movement of water during the inflow and outflow can provide a source of renewable energy to
communities living along the coast.
• Hence option (d) is the correct answer.
Q 77.D
• In our country, debris avalanches and landslides occur very frequently in the Himalayas. There are many
reasons for this. One, the Himalayas are tectonically active. They are mostly made up of sedimentary
rocks and unconsolidated and semi-consolidated deposits. The slopes are very steep. Hence statement 1
is correct and statement 2 is correct.
• Compared to the Himalayas, the Nilgiris bordering Tamilnadu, Karnataka, Kerala and the Western
Ghats along the west coast are relatively tectonically stable and are mostly made up of very hard
rocks; but, still, debris avalanches and landslides occur though not as frequently as in the Himalayas, in
these hills because:
o many slopes are steeper with almost vertical cliffs and escarpments in the Western Ghats and Nilgiris.
o mechanical weathering due to temperature changes and ranges is pronounced.
o they receive heavy amounts of rainfall over short periods. So, there is almost direct rock fall
quite frequently in these places along with landslides and debris avalanches. Hence statement 3
is correct.
Q 78.B
• Statement 1 is not correct: Black soil covers most of the Deccan Plateau which includes parts of
Maharashtra, Madhya Pradesh, Gujarat, Andhra Pradesh and some parts of Tamil Nadu.
• Statement 3 is correct: In the upper reaches of the Godavari and the Krishna, and the northwestern part
of the Deccan Plateau, the black soil is very deep. These soils are also known as the ‘Regur Soil’ or the
‘Black Cotton Soil’.
• Statement 2 is correct: The black soils are generally clayey, deep and impermeable. They swell and
become sticky when wet and shrink when dried. So, during the dry season, these soil develop wide cracks.
Thus, there occurs a kind of ‘self ploughing’. Because of this character of slow absorption and loss of
moisture, the black soil retains the moisture for a very long time, which helps the crops, especially, the
rain fed ones, to sustain even during the dry season.
• Chemically, the black soils are rich in lime, iron, magnesia and alumina. They also contain potash. But
they lack in phosphorous, nitrogen and organic matter. The colour of the soil ranges from deep black to
grey.
Q 79.B
• A lake is a body of water of considerable size, localized in a basin, that is surrounded by land apart from a
river or other outlet that serves to feed or drain the lake.
• Various types of lakes include
o Lakes Formed by Earth Movement: Tectonic lakes, Rift valley lakes
o Lakes Formed by Glaciation: Cirque lakes or tarns, Rock-hollow lakes, Kettle lakes, Lakes due to
morainic damming of valleys
o Lakes Formed by Volcanic Activity: Crater and caldera lakes
o Lakes Formed by Erosion: Karst lakes and wind deflated lakes
o Lakes Formed by Deposition
o Manmade lakes
• A rift valley is formed when two blocks of earth move apart letting the ‘in-between’ block slide
downwards. Rift valleys are deep, narrow, and elongated. Hence the lakes formed along rift valleys are
also deep, narrow, and very long. E.g.: Lakes Tanganyika, Malawi
• Cirque is a hollow basin cut into a mountain ridge. It has a steep-sided slope on three sides, an open
end on one side, and a flat bottom. When the ice melts, the cirque may develop into a tarn lake.
• A kettle is a depression/hole in an outwash plain formed by retreating glaciers or draining floodwaters.
The kettles are formed as a result of blocks of dead ice left behind by retreating glaciers.
• Hence option (b) is the correct answer.

25 www.visionias.in ©Vision IAS


Q 80.D
• In the North India Ocean, there is a complete reversal of the direction of currents between summer
and winter, due to the changes of monsoon winds. In summer from June to October, when the dominant
wind is the South-West Monsoon, the currents are blown from a south-westerly direction as the South
West Monsoon Drift. This is reverse in winter beginning from December, when the North-East Monsoon
blows the currents from the north-east as the North-East Monsoon Drift. The currents of the North Indian
Ocean, demonstrate most convincingly the dominant effects of winds on the circulation of ocean
currents.
Q 81.C
• Since no one can reach the center of the earth and make observations or collect samples of the material, a
lot of information is obtained through direct and indirect observations.
• Direct Sources:
o The most easily available solid earth material is surface rock or the rocks we get from mining areas.
o Volcanic eruption forms another source of obtaining direct information
• Indirect Sources
o Analysis of properties of matter indirectly provides information about the interior.
o Knowing the total thickness of the earth, scientists have estimated the values of temperature, pressure
and the density of materials at different depths.
o Another source of information are the meteors that at times reach the earth.
o The other indirect sources include gravitation, magnetic field, and seismic activity.
Q 82.A
• The Pachmarhi Biosphere Reserve is located in the Satpura Range of Madhya Pradesh. The
Satpura range forms the watershed between the river Narmada to the north and river Tapi to the
south. UNESCO designated it a biosphere reserve in 2009. It includes three wildlife conservation units:
Bori Sanctuary, Pachmarhi Sanctuary and Satpura National Park. Hence, option A is correct.
• The Achanakmar- Amarkantak Biosphere Reserve is the most dramatic and ecologically diverse
landscape in the Chhattisgarh and Madhya Pradesh states of India. The area of this Biosphere Reserve is
considered to be one of the major watersheds of peninsular India. It separates the rivers that drain into the
Arabian Sea and the Bay of Bengal. The reserve is also the source of three major river systems: the
Narmada, the Johilla and the Son River. Maikal hill ranges together with Vindhya and Satpura lie within
the Achanakmar-Amarkantak Biosphere Reserve. UNESCO designated it a biosphere reserve in 2012.
• The Agasthyamalai Biosphere Reserve was Established in 2001 and includes 3,500.36 km² of which
1828 km² is in Kerala and 1672.36 km² is in Tamil Nadu. Agasthyamalai Biosphere Reserve became part
of World Network of Biosphere Reserves in 2016. Is also under UNESCO's world list of biosphere
reserve.
• Pench National Park: It is not a UNESCO designated Biosphere Reserve. It is located on the southern
part of Madhya Pradesh, which also shares a boundary with Maharashtra. This national park has its area
segregated in two divisions- a) Priyadarshini National Park and Mowgli Pench Sanctuary. The national
park is named after the river- Pench, which while flowing from north to south, divides the national park in
almost equal halves namely eastern and western halves.
• Similipal National Park: It is a national park and a tiger reserve in the Mayurbhanj district in the Indian
state of Odisha. It is part of the Mayurbhanj Elephant Reserve, which includes three protected areas —
Similipal Tiger Reserve, Hadgarh Wildlife Sanctuary and Kuldiha Wildlife Sanctuary. Simlipal National
Park derives its name from the abundance of red silk cotton trees growing in the area. The park is home to
Bengal tiger, Asian elephant, gaur, and chausingha. along with some of the beautiful waterfalls like
Joranda and Barehipani Falls. This protected area is part of the UNESCO World Network of Biosphere
Reserves since 2009.

Q 83.C
• Near the equator, the tropics receive the most rain on a consistent basis. As a result, the fresh water
falling into the ocean helps decrease the salinity of the surface water in that region. Hence salinity is
lower than the average 35 ppt (parts per thousand) in equatorial waters.
• The lower salinity water rests above the higher salinity dense water. Salinity, generally, increases with
depth and there is a distinct zone called the halocline, where salinity increases sharply. Other factors
being constant, increasing salinity of seawater causes its density to increase. High salinity seawater,
generally, sinks below the lower salinity water. This leads to stratification by salinity.
• Sea water contains large amount of dissolved mineral matter of which Sodium Chloride alone
constitutes more 77 percent. The other important compounds include magnesium, calcium, potassium
salts. Hence option (c) is the correct answer.
26 www.visionias.in ©Vision IAS
• The average salinity of the Indian Ocean is 35ppt. The low salinity trend is observed in the Bay of
Bengal due to influx of river water by the river Ganga. On the contrary, the Arabian Sea shows higher
salinity due to high evaporation and low influx of fresh water.
Q 84.D
• The word El-Nino means 'Child Christ', because this current appears around Christmas in December.
December is a summer month in Peru (Southern Hemisphere) and in almost every three to seven years,
this sets in warm currents off the coast of Peru in the eastern Pacific and affects the weather in many
places including India.
• El-Nino is merely an extension of the warm equatorial current which gets replaced temporarily by
cold Peruvian current or Humbolt current. This current increases the temperature of water on the
Peruvian coast by 10 degree Celcius. This results in:
o the distortion of equatorial atmospheric circulation.
o irregularities in the evaporation of sea water.
o reduction in the amount of planktons which further reduces the number of fish in the sea.
• El-Nino is used in India for forecasting long range monsoon rainfall. The El-Nino event has led to
delayed or total failure of monsoon over most parts of the country in 1990-91, 1998, 2006-07 and
2015-16.
• Hence, all the statements are correct.

Q 85.C
• Savannah grasslands are typified by tall grasses and short trees. Hence, it is misleading to call the
savannah 'tropical grassland', because trees are always present with the luxuriant tall grass. The terms
"Parkland" and "bush-veld" describe the landscape better.
• Savannah climate is confined within the tropics and is best developed in Sudan, hence its name the Sudan
Climate. It is a transitional type of climate found between the equatorial rainforests and hot deserts.
• It is marked by alternate wet and dry seasons similar to monsoon climate but has considerably less
annual rainfall. Also, there is no distinct rainy season like in the monsoon climate.
• Days are hot and nights are cold i.e., extreme diurnal range of temperature.
• The prevailing winds of the region are the Trade Winds, which bring rain to the coastal districts but
are relatively dry by the time they reach the continental interiors or the western coasts.
• In true savanna lands, the grass is tall and coarse. The elephant grass may attain a height of even 15 feet.
• Trees usually have broad trunks, with water-storing devices to survive through the prolonged drought
and are usually umbrella-shaped.
• The savanna is known as the ‘big game country’ as thousands of animals are trapped or killed each year
by people from all over the world.
• Tribes like Masai, Hausa are found in the savannahs.
• Hence option (c) is the correct answer.

Q 86.D
• Recent Context: China has shown interest in joining Arms Trade Treaty. The UN General Assembly
adopted a historic Arms Trade Treaty (ATT) on 2 April 2013. This is the first-ever global treaty to
establish international norms aiming to prevent conventional weapons from being used for war crimes and
human rights abuses, or being diverted for illegal use by criminals. In other words, it was the first legally
binding treaty to regulate the international trade in conventional arms. It regulates the international trade
in conventional arms – from small arms to battle tanks, combat aircraft and warships.
• The ATT does not place restrictions on the types or quantities of arms that may be bought, sold, or
possessed by states. It also does not impact a state’s domestic gun control laws or other firearm ownership
policies. Its objectives include:
o Establish the highest possible common international standards for regulating or improving the
regulation of the international trade in conventional arms.
o Prevent and eradicate the illicit trade in conventional arms and prevent their diversion.
• As of September 2019, 105 states have ratified or acceded to the ATT, including five of the world's
top 10 arms producers (the United Kingdom, France, Germany, Italy, and Spain). It has 102 states
parties (Lebanon, Maldives, Botswana, Canada, Haiti, Guinea-Bissau, Palau, Suriname being members for
whom treaty entered into force in 2019 after they signed and ratified it ) and an additional 33 signatories,
excluding the United States which on April 26, 2019 announced during Donald Trump's speech at an
annual NRA meeting that the United States will be withdrawing its signature on the treaty. He signed an
order to this effect on stage, Countries that neither signed nor ratified included Russia, China, India,
Iran, North Korea, Saudi Arabia and Syria.
27 www.visionias.in ©Vision IAS
Q 87.A
• Earthquake waves are basically of two types:
o Body waves: They are generated due to the release of energy at the focus and move in all directions
traveling through the body of the earth. Hence, the name body waves. There are two types of body
waves:
✓ P waves (Primary waves): P-waves move faster and are the first to arrive at the surface. These
are also called ‘primary waves’. The P-waves are similar to sound waves. They travel through
gaseous, liquid, and solid materials. P-waves vibrate parallel to the direction of the wave. This
exerts pressure on the material in the direction of the propagation. As a result, it creates density
differences in the material leading to stretching and squeezing of the material. Hence, statement
1 is correct.
✓ S-waves (secondary waves): S-waves arrive at the surface with some time lag. These are called
secondary waves. S-waves can travel only through solid materials. The direction of vibrations
of S-waves is perpendicular to the wave direction in the vertical plane. Hence, they create troughs
and crests in the material through which they pass. Surface waves are considered to be the most
damaging waves. Hence, statement 2 is not correct.
o Surface Waves: The body waves interact with the surface rocks and generate new set of waves called
surface waves. These waves move along the surface. The velocity of waves changes as they travel
through materials with different densities. The denser the material, the higher is the velocity. Their
direction also changes as they reflect or refract when coming across materials with different
densities. The surface waves are the last to report on seismograph. These waves are more destructive.
They cause displacement of rocks, and hence, the collapse of structures occurs.
Q 88.D
• The Yamuna, the westernmost tributary and the longest tributary of Ganga. Hence statement 1 is
correct.
• The Yamuna has its source in the Yamunotri glacier on the western slopes of the Banderpunch range
(6316 km) in Uttarakhand. Whereas the Ganga rises In the Gangotri glacier near Gurmukh. Hence
statement 2 is correct.
• The Yamuna defines the state borders between Himachal Pradesh and Uttarakhand, and between
Haryana, Delhi and Uttar Pradesh. Hence statement 3 is correct
• When the Yamuna reaches the Indo-Gangetic plain, it runs almost parallel to the Ganges, the two rivers
creating the Ganges-Yamuna Doab region. It joins the Ganga at Prayag (Allahabad).

Q 89.D
• Some of the volcanic landforms are:
o Batholiths: These are dome-shaped granitic bodies formed from the cooled portion of magma
chambers. They appear on the surface only after the denudational processes remove the overlying
materials. They cover large areas, and at times, assume depth that may be several km. Hence option
(d) is the correct answer.
o Dykes: are sheet intrusions that cut across layers, or into an unlayered mass. Dikes usually follow
zones of weakness in the rocks they travel through. Even if they came to lie horizontal, for example
by folding, as long as they cut host rock strata they are still dikes. The rocks in dikes can be both
pegmatites (very coarse-grained) and aplites (sugary fine-grained). Several dikes together can form a
dike swarm. – In scientific terms, a dike is a relatively narrow tabular discordant body which
propagates through rock perpendicular to the least principal stress.
o Lapolith: As and when the lava moves upwards, a portion of the same may tend to move in a
horizontal direction wherever it finds a weak plane. It may get rested in different forms. In case it
develops into a saucer shape, concave to the sky body, it is called lopolith.
o Phacolith: A wavy mass of intrusive rocks, at times, is found at the base of synclines or at the top of
the anticline in the folded igneous country. Such wavy materials have a definite conduit to source
beneath in the form of magma chambers (subsequently developed as batholiths). These are called the
phacoliths.

28 www.visionias.in ©Vision IAS


Q 90.A
• The Himalayan Mountains form the northern mountain region of India.
• They are the highest mountain ranges in the world.
• These mountain ranges start from Pamir Knot in the west and extend up to Purvanchal in the east.
• These are the youngest folded & loftiest mountain range of the world. Hence, statement 2 is correct.
• They are formed by tectonic forces (due to continent-continent collision between the Indian Plate
and the Eurasian Plate) & are 2400 Km in length. Hence, statement 1 is correct.
• They are of varying width → from 400 Km in Kashmir to 160 Km Arunachal Pradesh
• Altitudinal variations are greater in the eastern part than in the western part.
• Prominent Features → Highest peaks, Deep valleys & Gorges, Glaciers, etc.
• There are no volcanoes in the Himalayan region. Hence, statement 3 is correct.
Q 91.B
• Saser Kangri is a mountain in India. It is the highest peak in the Saser Muztagh, the easternmost
subrange of the Karakoram range. Sasir Kangri is located within Ladakh, the northernmost union territory
in India.
• Lali Ane: Recently, an organisation representing people of more than 30 villages under two circles of
Arunachal Pradesh’s East Kameng district has opposed a proposed high-altitude firing range of the Indian
Air Force (IAF). Lali Ane in Nyishi dialect (Lali – enormous, magnificent, or confluence of mountain
peak and Ane – Mother) means – Mother of Mountain and associated with many myths and legends. Lal
Ane, a flat valley spread over 21-km in length and 6.5-km in width in the Lal Ane mountain range, much
larger than famous Aptani plateau, is situated at an altitude of about 12,000 feet above sea level, and
located around 30-km from Mengio circle of Papum Pare district in Arunachal Pradesh. It has alpine
meadows and remains snow-capped from November to March.
• Chang Chenmo: The Karakorams consist of a group of parallel ranges with several spurs. Only the
central part is a monolithic range. The width of the system is about 150 miles (240 km); the length is
increased from 300 miles (500 km) to 500 miles (800 km) if its easternmost extension—the Chang
Chenmo (Chinese: Qiangchenmo) and Pangong ranges of the Plateau of Tibet—is included. Marsimik is a
mountain pass in the Chang-Chenmo Range situated 96km east of Leh. Recently, reports have emerged
about intrusions and ‘face-offs’ in the Galwan River on the north bank of Pangong Tso, and possibly at
Hot Springs in Chang Chenmo River valley, and at Demchok.
Q 92.C
• Recent Context: The Asian Infrastructure Investment Bank (AIIB) has approved a USD750-million loan
to India to assist the government to strengthen its response to the adverse impacts of the COVID-19
pandemic on millions of poor and vulnerable households. Cofinanced with the Asian Development Bank,
the budgetary support will go toward bolstering economic aid for businesses, including for the informal
sector, expanding social safety nets for the needy, and strengthening the country’s health care systems.
• The Asian Infrastructure Investment Bank (AIIB) is a multilateral development bank with a mission to
improve social and economic outcomes in Asia and beyond.
• Headquartered in Beijing, the bank commenced its operations in January 2016 and has now grown to
102 (82 members and 20 prospective members) approved members from around the world with India
as one of its founding members. Hence statement 1 is not correct and statement 3 is correct.
• So far, AIIB has only invested in one solar power project outside Asia, in Egypt. While 75% of the
capital is from Asia, several non-Asian regions like Europe, North America, some East African and
Latin American countries have also joined the bank as members. Hence statement 2 is not correct.
29 www.visionias.in ©Vision IAS
• Unlike most other multilateral development banks set up by advanced economies, AIIB is the first major
multilateral development bank where principal contributors are the borrowing members themselves. India
has been the largest borrower of Asian Infrastructure Investment Bank (AIIB) lending since the time the
Bank started its operations.
• The country with the biggest vote share in the institution is China (26.65 percent), followed by India (7.65
percent), Russia (6.06 percent; the country has not been given a single project as of yet), Germany (4.2
percent), and South Korea (3.54 percent).

Q 93.C
• A monsoon is a seasonal change in the direction of the prevailing, or strongest, winds of a region.
Monsoons cause wet and dry seasons throughout much of the tropics. Monsoons always blow from cold
to warm regions.
• Winter monsoons last from October to April. The dry winter monsoon blows from the northeast. These
winds start in the air above Mongolia and northwestern China.
• The Himalayas prevent much of the cool air from reaching places like southern India and Sri Lanka,
keeping them warm all year.
• They do not cause rainfall in most parts of India because:
o they have little humidity as they move from land to the sea
o due to anticyclonic circulation (high-pressure center) on land, the possibility of rainfall from them
reduces.
• So, most parts of India do not have rainfall in the winter season. Winter monsoons are sometimes
associated with droughts.
Q 94.B
• The Hot, Wet Equatorial Climate, is a climate of great uniformity of temperature throughout the year. It
is found between 5 degrees and 10 degrees north and south of the equator, in the lowlands of Amazon, the
west coast of Africa, Malaysia, and the Indonesian islands. Though the precipitation is well distributed
throughout the year, there are two periods of maximum rainfall, which occur shortly after the equinoxes.
Precipitation is mainly of convectional type due to intense evaporation.
• The Tropical Marine Climate is experienced along the eastern coast of tropical lands in Central
America, West Indies, north-eastern Australia, eastern Africa, etc. They receive steady rainfall from the
Trade Winds all the time. The rainfall is both orographic where the moist trades meet upland
masses as in eastern Brazil, and convectional due to intense heating during the day and in summer.
There is no month without any rainfall. Hence option (b) is the correct answer.
• The Tropical Monsoon Climate is experienced where there are seasonal winds with on-shore wet
monsoons in the summer and off-shore dry monsoons in the winter. They are best developed in the Indian
sub-continent, south-east Asia and northern Australia.
• The Sudan Climate is characterized by the alternate hot, rainy season and cool, dry season. This
transitional type of climate is found between the equatorial forests and the trade wind hot deserts.

Q 95.B
• There is two distinct systems of monsoons in Indian.
• Southwest Monsoons:
o It causes rainfall during the months of June to September.
o It burst in the first week of June in the coastal areas of Kerala, Karnataka, Goa, and Maharashtra while
in the interior parts of the country, it may be delayed to the first week of July.
o It is responsible for the majority of rainfall in North as well as South India. It causes rainfall in
Western Ghats, Western Coastal Plain, Gujarat region, Maharashtra coast, Central India, Chotanagpur
plateau, North Eastern region and Gangetic plains.
• North-East or Winter Monsoon or Retreating Monsoon
o It causes rainfall in the months of October to December.
o It is mainly responsible for rainfall on the eastern coast of Tamil Nadu and a small adjoining region.
Hence, statement 1 is not correct.
• Northern India experiences a high daily and annual range of temperatures than Southern India.
o The northern part of India lies in the sub-tropical and temperate zone and the part lying south of the
Tropic of Cancer falls in the tropical zone. The tropical zone being nearer to the equator experiences
high temperatures throughout the year with a small daily and annual range. The area north of the
Tropic of Cancer being away from the equator experiences an extreme climate with a high daily and
annual range of temperature. Hence, statement 2 is correct.

30 www.visionias.in ©Vision IAS


Q 96.C
• Patagonian desert is also known as the Patagonian Steppe is the largest desert in Argentina. It is
located primarily in Argentina and is bounded by the Andes, to its west, and the Atlantic Ocean to its east,
in the region of Patagonia, southern Argentina. To the north the desert grades into the Cuyo Region and
the Pampas. The central parts of the steppe are dominated by shrubby and herbaceous plant species albeit
to the west, where precipitation is higher, bushes are replaced by grasses. Topographically the deserts
consist of alternating tablelands and massifs dissected by river valleys and canyons. The more western
parts of the steppe host lakes of glacial origin and grades into barren mountains or cold temperate forests
along valleys. Hence, pair 1 is not correctly matched.
• Mojave desert is an arid rain-shadow desert and the driest desert in North America. It is in the
Southwestern United States, primarily within southeastern California and southern Nevada. It extends
from the Sierra Nevada range to the Colorado Plateau and merges with the Great Basin to the north
and the Sonoran Desert to the south. Hence, pair 2 is correctly matched.
• Gibson desert is the arid zone in the interior of Western Australia. Hence, pair 3 is not correctly
matched. The desert lies south of the Tropic of Capricorn between the Great Sandy Desert (north),
the Great Victoria Desert (south). It is known for its undulating red dunes, desert pavement, and sandy
plains and the desert grass Triodia. It is a fire-prone landscape.

Q 97.C
• Lithification
o Rocks (igneous, sedimentary and metamorphic) of the earth’s surface are exposed to various
denudational agents and are broken up into various sizes of fragments. Such fragments are transported
by different exogenous agencies and deposited. These deposits through compaction turn into
rocks. Hence option (c) is correct.
o Lithification is a process of porosity destruction through compaction and fermentation.
o In many sedimentary rocks, the layers of deposits retain their characteristics even after lithification.
Hence, we see a number of layers of varying thickness in sedimentary rocks like sandstone, shale etc.
• Recrystallization
o It is a metamorphic process that occurs under temperature and pressure where atoms of a mineral are
reorganized by diffusion and/or dislocation glide. The mineral composition may remain unchanged.
o Limestone is a sedimentary rock that undergoes metamorphic recrystallization to form marble,
and clays can recrystallize to muscovite mica.
• Foliation
o It refers to repetitive layering in metamorphic rocks. Each layer can be as thin as a sheet of paper, or
over a meter in thickness.
o The word comes from the Latin folium, meaning "leaf", and refers to the sheet-like planar structure.
o It is caused by shearing forces or differential pressure.
• Exfoliation
o It describes the peeling away of sheets of rock millimeters to meters in thickness from a rock's surface
due to a range of physical and chemical processes during exhumation and weathering.
31 www.visionias.in ©Vision IAS
Q 98.D
• The Ten Degree Channel is a channel that separates the Andaman Islands and Nicobar Islands
from each other in the Bay of Bengal. Hence pair 1 is correctly matched. The two sets of islands
together form the Indian Union Territory (UT) of Andaman and Nicobar Islands.
• The Coco Strait is between the North Andaman Islands and the Coco Islands of Myanmar. Hence
Pair 3 is not correctly matched.
• South Andaman (Great Andaman) and Little Andaman are separated by Duncan Passage. Hence
pair 2 is not correctly matched.
• The Grand Channel is between the Great Nicobar Islands and the Sumatra islands of Indonesia.

Q 99.C
• Residual Mountains:
o The weathering and different agents of erosion – rivers, winds, glaciers, etc. are constantly acting on
the earth’s crust.
o This process of erosion depends on the shape and structure of the rocks.
o Due to the agents of erosion, the softer rocks of the upland are eroded much more rapidly than the
areas of harder rock.
o After thousands of years, soft rocks are worn down into the sand and the hard rocks are left standing
up in the area that has been reduced in height. These are called residual mountains.
o The residual mountains are also called relict mountains.
o Examples of Residual Mountains in India are the Nallamala hills, Mahendragiri hills, the Aravalli
hills, the Javadi hills, the Veliconda hills, and the Palkonda range.
• Hence option (c) is the correct answer.

Q 100.B
• The Inter Tropical Convergence Zone (ITCZ) is a low pressure zone located at the equator where trade
winds converge, and so, it is a zone where air tends to ascend. Hence, statement 1 is not correct.
• The location of the ITCZ gradually varies with the seasons.
• In July, the ITCZ is located around 20°N-25°N latitudes (over the Gangetic plain), sometimes called the
monsoon trough. This monsoon trough encourages the development of thermal low over north and
northwest India. Due to the shift of ITCZ, the trade winds of the southern hemisphere cross the equator
between 40° and 60°E longitudes and start blowing from southwest to northeast due to the Coriolis force.
It becomes southwest monsoon.
• In winter, the ITCZ moves southward, and so the reversal of winds from northeast to south and southwest,
takes place. They are called northeast monsoons.
• As the axis of the monsoon trough oscillates, there are fluctuations in the track and direction of these
depressions, and the intensity and the amount of rainfall vary from year to year. Hence, statement 2 is
correct.

32 www.visionias.in ©Vision IAS

You might also like